• Shuffle
    Toggle On
    Toggle Off
  • Alphabetize
    Toggle On
    Toggle Off
  • Front First
    Toggle On
    Toggle Off
  • Both Sides
    Toggle On
    Toggle Off
  • Read
    Toggle On
    Toggle Off
Reading...
Front

Card Range To Study

through

image

Play button

image

Play button

image

Progress

1/200

Click to flip

Use LEFT and RIGHT arrow keys to navigate between flashcards;

Use UP and DOWN arrow keys to flip the card;

H to show hint;

A reads text to speech;

200 Cards in this Set

  • Front
  • Back
Ethics and Professional Practice
1. A psychologist in a rural area is referred a 17-year-old male who has been abusing OxyContin. Although the psychologist is trained and experienced in the treatment of substance abuse, she has not received training in, nor experience with, OxyContin abuse. The psychologist should:
A refer the patient to a psychologist outside the area who is experienced in the treatment of OxyContin abuse
B agree to treat the patient after attending a workshop on the treatment of OxyContin abuse
C refer the patient for inpatient detoxification
D provide the patient with treatment and read relevant literature
ANSWER "D"
Psychologists have an ethical responsibility to practice within the boundaries of their competence. However, according to APA's Ethics Standard 2.01(d), "When psychologists are asked to provide services to individuals for whom appropriate mental health services are not available and for which psychologists have not obtained the competence necessary, psychologists with closely related prior training or experience may provide such services in order to ensure that services are not denied if they make a reasonable effort to obtain the competence required by using relevant research, training, consultation, or study." It should not, therefore, be necessary to refer the patient to a provider out of the area. It also would not be in the patient's best interest to postpone treatment until the psychologist can attend a workshop on the subject (B). And, although OxyContin abuse is serious and potentially fatal, it generally would not require inpatient detoxification (C) unless the abuse progressed to dependence.
Developmental Psychology
2. The realization that gender is unaffected by superficial changes in appearance or activity is referred to as:
A gender constancy
B gender stability
C gender identity
D ego-dystonic transvestism
ANSWER "A"
Gender concept develops in a predictable sequence of stages during childhood. The first stage is known as "gender identity," which is the ability to categorize self and others as male or female. There is evidence that gender identity develops as early as 9 months or as late as 3 years, depending on how the researchers define it. The next stage, which develops by 4 years of age, is "gender stability," which is an understanding that one's gender does not change over time. The final stage is known as "gender constancy," which is characterized by an understanding that gender stays the same despite changes in appearance. Gender constancy is achieved by age 5 or 6.
Diagnosis and Psychopathology
3. Research on gender differences in anxiety has most often found that:
A females have more general anxiety than males
B males have more general anxiety than females
C females have more social and general anxiety than males
D males have more social and general anxiety than females
ANSWER "A"
The results of a meta-analysis on gender differences concluded that females (across all age groups) have a higher level of general anxiety than males. Males scored very slightly higher than females in level of social anxiety, although this difference was not significant (A. Feingold, Gender Differences in Personality: A Meta-Analysis. Psychological Bulletin, 1994, 116(3), 429-456).
Industrial/Organizational Psychology
4. Which of the following is NOT considered a disability according to the Americans with Disabilities Act?
A Gender Identity Disorder
B Paraplegia
C Schizophrenia
D Generalized Anxiety Disorder
ANSWER "A"
The Americans with Disabilities Act (ADA) of 1990 prohibits employers from discriminating against a qualified individual based on his or her disability. The ADA defines a disability as "a physical or mental impairment that substantially limits one or more of the major life activities" of an individual. However, it specifically excludes certain conditions including: transvestism, transsexualism, pedophilia, exhibitionism, voyeurism, gender identity disorders not resulting from physical impairments, or other sexual behavior disorders, compulsive gambling, kleptomania, pyromania, and psychoactive substance use disorders resulting from current illegal use of drugs. All of the other choices would, therefore, be considered a disability.
Neuropsychology
5. Recent findings indicate there is a type of cells in the premotor cortex which are activated when a person executes object-directed actions of a certain general type, and when the individual sees another individual performing actions of the same type. These cells are called:
A imitation neurons
B experiential neurons
C mirror neurons
D simulation neurons
ANSWER "C"
C. Mirror neurons are a class of neurons that are activated by action observation, or when an individual witnesses actions performed by others (e.g., observing someone being hit by a ball), the individual’s brain simulates as if it is performing the same specific actions (e.g., being hit by a ball). This experiential insight or “gut-feeling” is proposed to be due to neural underpinnings. Specifically, an embodied simulation - which is an unconscious, automatic, and pre-reflexive functional mechanism for the modeling of objects, agents, and events. Action observation differs from motor imagery. In motor imagery, an internal event or deliberate act of will triggers the simulation process, whereas in action observation it is an external event. While there are different patterns of brain activation, there is a commonality in the activation of parietal and premotor cortical networks in both simulations of action. Current research is investigating the role the mirror neuron system plays in empathy, Autism and Schizophrenia. (See: Gallese, V. and Goldman, A. (1998). Mirror neurons and the simulation theory of mind-reading. Trends in Cognitive Sciences, 12, 493-501. and Rizzolatti, G., Fadiga, L., Gallese, V. and Fogassi, L. (1996). Premotor cortex and the recognition of motor actions. Cognitive Brain Research, 3, 131-141. and Rizzolatti, G. and Craighero, L. (2004). The mirror neuron system. Annual Review of Neuroscience, 27, 169-192.).
Neuropsychology
6. Which of the following is the most frequently used scale to measure orientation?
A Galveston Orientation and Amnesia Test
B Gollingberg Orientation and Awareness Test
C Gross Orientation and Awareness Test
D Global Orientation and Amnesia Test
ANSWER "A"
A. The Galveston Orientation and Amnesia Test (GOAT), which assesses temporal orientation primarily, was developed to serially evaluate cognition during the subacute stage of recovery from closed head injury. The scale measures orientation to person, place, and time, and memory for events preceding and following the injury. (See: Levin, H.S., O'Donnell, V.M., & Grossman, R.G. (1975). The Galveston orientation and amnesia test: A practical scale to assess cognition after head injury. Journal of Nervous and Mental Diseases, 167, 675-684.) .
Community Psychology
7. Research has identified the following factors as barriers that reduce the likelihood that a woman will seek or remain in substance abuse treatment. Of the following, which has been most often cited as a primary barrier to substance-abuse treatment for women:
A a belief that treatment is ineffective
B denial of a substance abuse problem
C child and childcare concerns
D sexual harassment from male counselors
ANSWER "C"
C. Only one of the responses is consistently cited by studies as a primary or most frequently mentioned barrier and that is factors related to children. Research has confirmed that women with substance abuse problems are more likely than men to be caring for dependent children. Many women are concerned about custody issues; others are concerned about childcare while they receive treatment. According to a recent United Nations report on substance abuse treatment, “lack of childcare is probably the most consistent factor restricting women’s treatment access identified in the literature” [United Nations Office on Drugs and Crime, Substance Abuse Treatment and Care for Women: Case Studies and Lessons Learned, New York, United Nations, 2004, p. 18]. While sexual harassment (d.) of women has been identified as a problem at some treatment centers and some women may be deterred by concerns over treatment effectiveness (a.), neither has been identified as a primary barrier to treatment. Women are more likely to express shame and guilt about their substance abuse than to deny that they have an abuse problem (b.).
Diagnosis and Psychopathology
8. A person with Somatization Disorder is most likely to also have which of the following diagnoses:
A a Substance Abuse Disorder
B a Personality Disorder
C Schizophrenia
D Panic Disorder
ANSWER "B"
Somatization Disorder has several frequently occurring comorbid conditions. Research indicates that 61% of Somatization Disorder patients have one or more co-occurring personality disorders. The next most frequent co-diagnoses are Major Depression (55%), Generalized Anxiety Disorder (34%), and panic disorder (26%). Substance-Related Disorders are also frequently associated with Somatization Disorder, however, not as frequently as the above diagnoses. [See: G.O. Glabbard, Treatment of Psychiatric Disorders, Volume 2 (2nd ed.), Washington, DC, American Psychiatric Press, 1995].
Ethics and Professional Practice
9. You see a client for the first time. She is already receiving treatment from another professional, and she is not clear about why she wants additional treatment from you. In this situation, you should
A treat the client and let her discuss what she is doing with the other professional when she is comfortable enough to do so.
B contact the other professional and attempt to work out a coordinated treatment plan with him or her.
C attempt to discuss with the client the reason she is seeking the services of a second therapist.
D inform the client that it would be unethical for you to continue therapy with her.
ANSWER "C"
In answering ethics questions, it is a good idea to bring to mind the language of the applicable ethical standard and choose the answer that is most consistent with that standard. According to Standard 10.04 (Providing Therapy to Those Served by Others), psychologists should consider treatment issues and the client's welfare, and discuss these issues with the client, “in order to minimize the risk of confusion and conflict,” and should “proceed with caution and sensitivity to the therapeutic issues.” Of the choices listed, only C states that you would discuss the issue with the client, and is therefore the best answer.
Neuropsychology
10. A person who doesn't feel something they are touching probably has damage to:
A Parietal lobe
B Occipital lobe
C Frontal lobe
D Temporal lobe
ANSWER "A"
The parietal lobe contains the somatosensory cortex. Its functions include the processing of touch-pressure, temperature, kinesthesia and pain.
Industrial/Organizational Psychology
11. In order to decrease job turnover, an interviewer should:
A Rely on biodata
B Rely on cognitive testing
C Hire only older people
D Rely on an interest inventory
ANSWER "A"
Cognitive tests are the most valid predictors of job performance across jobs and settings–and job performance would include job continuity. However, biodata is particularily useful for predicting turnover. In the specific area of predicting turnover, biodata has been found to have validity coefficients of .77 and .79 for predicting the turnover of whites and African-Americans respectively (Cascio 1976). Hunter and Hunter (1984) report an average validity coefficient of .53 when cognitive tests are used to predict performance ratings and .75 when they are used to predict performance on a work sample.
Diagnosis and Psychopathology
12. According to current research, the best predictor(s) for alcoholism would be
A family history of alcoholism.
B environmental stresses and opportunities for observational learning.
C interpersonal pressure and identifications.
D age and SES.
ANSWER "A"
If you wanted to find the best single predictor, you'd find out about alcoholism in the natural relatives of the patient. Even if the person is adopted away from the natural parents, the genetic connection is still the strongest one we have. So, when doing an initial assessment, you could ask about alcohol/drug abuse among family members. By the way, a good guess in answering any question structured as "the best predictor of (some disorder)" is "family history of that disorder." You won't be right 100% of the time, but you will be the majority of the time.
Ethics and Professional Practice
13. You have been seeing a client for several months and believe that treatment has been a success. When you suggest termination to the client, he expresses a desire to continue therapy. Both you and the client are unable to come up with additional treatment goals; however, the client states that he really looks forward to coming to therapy just to talk. In this situation, you should:
A terminate treatment immediately.
B terminate treatment after a period of pretermination counseling, in which the client's reasons for not wanting to terminate are explored.
C continue seeing the client until he is ready to end therapy.
D seek consultation.
ANSWER "B"
Ethical standard 10.10 requires that a psychologist terminate a professional relationship when it becomes “reasonably clear that the client/patient no longer needs the service, is not likely to benefit, or is being harmed by continued service." Prior to termination, the psychologist must “provide pretermination counseling and suggest alternative service providers as appropriate.".
Ethics and Professional Practice
14. A psychologist sends the MMPI to a client through the mail. The client will complete the test at home and return it to the psychologist by mail. This is:
A acceptable as long as the client has signed a statement saying she will not breach test security.
B acceptable as long as the psychologist provides adequate instructions over the phone.
C acceptable as long as the test is sent by certified mail.
D unacceptable.
ANSWER "D"
This is pretty clear: As a psychologist, you're responsible for maintaining test security. In addition, you'd want to administer the MMPI under controlled conditions to obtain valid results. If you sent the test to the client, you'd have no control over testing conditions.
Neuropsychology
15. Fluoxetine is LEAST likely to cause:
A anxiety
B confusion
C headache
D nausea
ANSWER "B"
Like all medications, fluoxetine (Prozac) has a risk of side-effects. The most commonly observed side effects of Prozac are: nausea, headache, insomnia, anxiety, and somnolence. Confusion is not one of the common side effects.
Clinical Psychology
16. Research on the use of mental health services by members of minority groups indicates that:
A utilization rates are lower for members of all minority groups than for whites.
B utilization rates are higher for members of all minority groups than for whites.
C utilization rates are higher for members of some minority groups and lower for members of other minority groups than for whites.
D utilization rates for members of minority groups are about the same as the rates for whites.
ANSWER "C"
It's difficult to draw any general conclusions about utilization rates because the research findings are inconsistent. However, a 1991 study by Sue et al. (which is frequently cited in the literature) reports underutilization by Asian-and Latino-Americans and overutilization by African-Americans. The results of other studies also suggest that there are group differences in utilization rates, so response C is the best.
Developmental Psychology
17. Symptoms of separation anxiety usually begin to appear at approximately:
A 5 months
B 9 months
C 12 months
D 18 months
ANSWER "B"
Although separation anxiety sometimes begins as early as 5 or 6 months, it typically has its onset at 8 or 9 months of age, rises dramatically until the age of 18 months, and then gradually falls off until it becomes negligible between the ages of 24-36 months. Prior to the onset of separation anxiety, babies do not typically protest when separated from their primary caretakers, even though a number of attachment behaviors develop during this time.
Developmental Psychology
18. According to Jerry Patterson and associates the families of highly aggressive boys are characterized by:
A coercive interactions and poor parental monitoring
B Coercive interactions and high parental monitoring
C Low interaction and low parental monitoring
D Excessive television viewing
ANSWER "A"
Patterson, Chamberlin and Reid 1982 "A Comparative evaluation of a parent-training program" Behavior Therapy, 13(5) 638-650 proposed a coercive family interaction model. It hypothesized that children first learn aggressive behaviors from their parents. These parents also use harsh discipline and reward their children's aggressiveness with attention and approval. As a result, the aggressiveness of the parent-child interaction continues to escalate.
Clinical Psychology
19. According to Beck, a depressed man is most likely to believe that:
A the world is unfair, his future is hopeless, but he is a good person
B he is worthless, his future is hopeless, but the world is just
C he is worthless, his future is hopeless, and the world is unfair
D he is worthless, the world is unfair, but his future is hopeful
ANSWER "C"
Depressed people tend to distort their perceptions and interpret events from a negative perspective. Beck referred to the "cognitive triad" which consists of negative thoughts about the self, future, and the world. Choice C best represents the cognitive triad.
Developmental Psychology
20. The cognitive outcomes of malnutrition during the final months of prenatal development are severe because:
A malnutrition causes degeneration of existing neurons.
B malnutrition interferes with the development of new neurons and the connections between neurons.
C malnutrition reduces the production of certain neurotransmitters.
D malnutrition leads to the development of deformed, nonfunctional neurons.
ANSWER "B"
The brain is the last organ to develop, which means that the last few months of prenatal development are most critical. Malnutrition during the final months can lead to irreversible damage due to the fact that it interferes with the development of new neurons and dendrites, which connect the neurons.
Diagnosis and Psychopathology
21. Individuals with Major Depressive Disorders who experience abnormalities in the sleep cycle may have any of the following sleep disturbances EXCEPT:
A early morning awakening
B sleep continuity decrease
C REM latency decrease
D slow-wave sleep increase
ANSWER "D"
D. Depression is associated with decreased slow-wave or non-REM sleep as well as, early morning waking (a.), decreased sleep continuity (b.) and earlier onset of REM sleep or decreased REM latency (c.). .
Clinical Psychology
22. Research by Sue and his colleagues (1991) suggests that which of the following clients is most likely to return for a second session of psychotherapy?
A an African-American client
B a Latino-American client
C an Anglo-American client
D an Asian-American client
ANSWER "D"
Studies on therapy dropout rates have produced inconsistent results. This question is asking about a particular study, however (Sue et al., 1991), which found that African-Americans have the highest dropout rates, while Asian-Americans have the lowest dropout rates.
Ethics and Professional Practice
23. The mother of a 3-year old has been told her daughter is very gifted. She wants to encourage the child's abilities and asks you, a school psychologist, to assess the girl's IQ score level and suggest a plan for her future education. You should:
A comply with the request.
B comply with the request, but inform the mother that test results at this age are not very valid as predictors of future performance.
C suggest that the test not be given, but agree to help the mother in deciding on the future educational plans for her child.
D inform the mother that there are no standardized valid tests available for children of such a young age.
ANSWER "B"
In this case there is nothing wrong with you testing the child and tests are available. But you should also use your knowledge and judgment to counsel the mother about the use of tests for such a young child. While it's true that there is some predictive validity for intelligence tests starting at about 18 months to 2 years of age, you should still tell the mother that many things will determine her daughter's future school performance and that we can't be so sure she will continue as she presently is.
Diagnosis and Psychopathology
24. A person unwilling to get involved with others unless certain of being liked, is inhibited as a result, and is unusually reluctant to take personal risks, would most likeley be diagnosed with:
A Schzoid Personality Disorder
B Dependent Personality Disorder
C Avoidant Personality Disorder
D Schizotypal Personaltiy Disorder
ANSWER "C"
An Avoidant Personality is described by DSM-IV as one with "a pervasive pattern of social inhibition, feelings of inadequacy and hypersensitivity to negative evaluation. While both Schzoid and Schizotypal Personalities are also characterized by social isolation, individuals with Avoidant Personalities want to be liked, and feel their isolation deeply.
Diagnosis and Psychopathology
25. Although Jasper has worked for the same company for nearly 15 years, he has no friends at work and never eats lunch or takes breaks with his coworkers. Jasper hasn't been to a party for eight years. He says he'd like to go to parties but he feels no one will want to talk with him and that people will make fun of him. He also says that he often feels lonely. The best diagnosis is
A Schizoid Personality Disorder.
B Paranoid Personality Disorder.
C Avoidant Personality Disorder.
D Schizotypal Personality Disorder.
ANSWER "C"
The combination of social avoidance, fear of humiliation, and loneliness are characteristic of Avoidant Personality Disorder.
Learning Theory and Behavior Therapy
26. In a study, subjects are asked to memorize a list of 12 unrelated words. After a brief period of time, they are asked to count backwards from 100 and are then tested on the word list. These subjects are
A the experimental subjects in a study on retroactive interference.
B the control subjects in a study on retroactive interference.
C the experimental subjects in a study on proactive interference.
D the control subjects in a study on proactive interference.
ANSWER "B"
Knowing the difference between retroactive and proactive interference would have helped you narrow the choices down to responses A and B. (In retroactive interference subsequent learning interferes with previous learning). In studies on retroactive interference, experimental subjects learn material similar to the original list, while control subjects engage in a dissimilar activity (e.g., counting backwards) that keeps them from rehearsing the list.
Developmental Psychology
27. Piaget believed that children are "spontaneous liars" and considered their false statements to be natural and harmless until about:
A Age 3 or 4
B Age 4 or 5
C Age 5 or 6
D Age 7 or 8
ANSWER "D"
Piaget thought that around age 7 or 8 children begin to intentionally communicate false statements. Subsequent research suggest that children as young as age four lie intentionally, most often to avoid punishment or obtain a reward. (Ceci S.J., Leichtman M., and Putnick, M., 1992 Cognitive and Social Factors in Early Deception Hillsdale NJ Erlbaum).
Diagnosis and Psychopathology
28. According to DSM-IV, which of the following types of dementia is most common among those over the age of 65?
A Vascular Dementia
B Alzheimer's Type
C Substance-Induced Persisting Dementia
D Dementia due to Head Trauma
ANSWER "B"
Dementia of the Alzheimer's Type is the most common type of dementia among older adults, and the rate of Alzheimer's increases with increasing age. About 20% of people over the age of 85 have this disorder.
Ethics and Professional Practice
29. Which of the following infractions accounts for the largest percentage of malpractice costs?
A lack of competence
B breach of confidentiality
C sexual contact with clients
D improper billing practices
ANSWER "C"
The infraction that accounts for the greatest proportion of malpractice claims depends on how the infractions are categorized. However, regardless of the way that malpractice claims are grouped, sex with clients is always the infraction that accounts for the greatest insurance costs, with reported figures ranging from 45 to 50%.
Test Construction
30. A person obtains a raw score of 70 on a Math test with a mean of 50 and an SD of 10; a percentile rank of 84 on a History test; and a T-score of 65 on an English test. What is the relative order of each of these scores?
A History >> Math >> English
B Math >> History >> English
C History >> English >> Math
D Math >> English >> History
ANSWER "D"
Before we can compare different forms of scores, we must transform them into some form of standardized measure. A Math test which has a mean of 50 and an SD of 10 indicates that a raw score of 70 would fall 2 standard deviations above the mean. Assuming a normal distribution of scores, a percentile rank of 84 on a History test is equivalent to 1 standard deviation above the mean. If you haven't memorized that, you could still figure it out: Remember that 50% of all scores in a normal distribution fall below the mean and 50% fall above the mean. And 68% of scores fall within +/- 1 SD of the mean. If you divide 68% by 2 - you get 34% (the percentage of scores that fall between 0 and +1 SD). If you then add that 34% to the 50% that fall below the mean - you get a percentile rank of 84. Thus, the 84 percentile score is equivalent to 1 SD above the mean. Finally, looking at the T-score on the English test - we know that T-scores always have a mean of 50 and an SD of 10. Thus a T-score of 65 is equivalent to 1½ standard deviations above the mean. Comparing the 3 test scores we find the highest score was in Math at 2 Sds above the mean, followed by English at 1½ SDs above the mean, and History at 1 SD above the mean.
Social Psychology
31. When a group of people is attempting to develop solutions to problems using brainstorming, the resulting ideas, as compared to those generated by individuals working alone, are
A fewer and of lower quality.
B greater and of better quality.
C fewer but of better quality.
D greater but not necessarily of better quality.
ANSWER "A"
This question references research about brainstorming, a technique in which people in a group are encouraged to generate all the ideas they can, without worrying about evaluation or censure. This technique is generally not too effective; it typically produces fewer ideas and the quality is poorer.
Learning Theory and Behavior Therapy
32. Which of the following best describes the distinction between shaping and chaining?
A shaping involves reinforcement for successive approximations of a single behavior; chaining involves providing a chain of reinforcement for one behavior
B shaping involves providing successive approximations of a reinforcer for a given behavior over a period of time; chaining involves reinforcing multiple behaviors with the same reinforcers
C shaping involves reinforcing component parts of one simple behavior; chaining involves many simple behaviors that are linked to form a more complex behavior
D there is no distinction between these two terms; they are essentially synonymous
ANSWER "C"
Choice C best describes the distinction between shaping and chaining. Shaping involves reinforcing successive approximations of a single behavior as the person approaches that behavior. For instance, an autistic individual learning to speak might be reinforced first for moving his mouth, then for uttering nonsense sounds, and then for saying a particular word. The successive approximations being reinforced are all components of one behavior. Chaining, on the other hand, involves linking a group of simple behaviors to form a more complex response chain. For instance, a child learning to put on his shirt might first learn to open the drawer, then put his shirt on over his head, and then button the shirt. Each of these behaviors is a separate behavior, but can be linked as a set to constitute the more complex behavior of putting on the shirt.
Clinical Psychology
33. A child feels confident in reaching out to others because as a baby they had a secure bond with their mother. This information would be important to a psychologist with the theoretical approach of:
A cognitive- behavioral
B object-relations
C Family systems
D Psychotherapeutic
ANSWER "B"
According to object relations theory, an object introject is the mental representation of a person, either the self or the mother. In a healthy environment the child comes to develop representations of itself based on the early interactions with the mother. When normal and age-appropriate development is delayed or skewed by having a distrubed parent, the person could easily fail to develop object introjects.
Diagnosis and Psychopathology
34. All of the following statements are true of Tourette's Disorder, except:
A onset of the disorder always occurs before the age of 18.
B both motor tics and vocal tics are present.
C uncontrollable outbursts of vulgar and obscene utterances are present in the majority of cases.
D the disorder is more common in males than females.
ANSWER "C"
Tourette's Disorder is a Tic Disorder that involves multiple motor tics (involuntary jerky movements) and vocal tics (vocal sounds such as grunts, barks, and clicks). Coprolalia, or involuntary utterances of obscenities, occurs in 10% or less of cases -- not the majority, which is why C is correct (remember, the question is asking for the statement that isn't true). For a diagnosis to be made, onset of the disorder must be before age 18. The disorder is between 1.5 and 3 times more common in males than in females.
Ethics and Professional Practice
35. A psychologist is undertaking a research project in a high security prison on the effects of certain food additives on behavior. The prison population is surveyed, informed of the purposes of the study, and volunteers are sought. The volunteer participants are again explained the purpose of the study and are asked to give their written consent to be participants. Because of the nature of the study, its importance to the food industry and the governmental regulatory agency, and the need to keep as much control of the participants' food intake as is feasible, the participants are informed that, once they begin the study, they are expected to remain in it at least until the first evaluation period, six weeks from the beginning of the research program. They are told that they shouldn't agree to participate unless they are reasonably sure they can complete at least this initial phase. Those participants who then agree and give written consent are included in the study. According to APA's Ethical Principles, such a study:
A should not be undertaken as the participants are not allowed to freely withdraw.
B would be allowed only if the participants gave specific acknowledgment of their agreement to abide by the withdrawal arrangements.
C is probably unethical because the prison research review committee did not also agree to the participation arrangements.
D meets all research guidelines and can be undertaken.
ANSWER "A"
The ethical standards require that research participants be allowed to withdraw from a study at any time. Hence the answer must be A.
Learning Theory and Behavior Therapy
36. Memory for the rules of logic and inference is part of
A procedural memory.
B semantic memory.
C episodic memory.
D read-only memory
ANSWER "B"
Long-term memory has been divided into three components: semantic memory, procedural memory, and episodic memory. Semantic memory includes memory for the rules of logic and inference, as well as knowledge about language (e.g., what words mean and how they are used). Procedural memory includes information about how to do things, such as how to drive a car. Episodic memory contains information about events that have been personally experienced. Read-only memory (ROM) is not part of long-term memory -- it is a computer term.
School Psychology/Intelligence
37. The concordance rate for IQ between children and adopted parents is around
A .17.
B .33.
C .50.
D .60.
ANSWER "A"
The concordance rate for unrelated people living in a similar environment, into which category this question falls, is around .17. As the genetics get more similar, of course the rate increases, as you'd expect.
Ethics and Professional Practice
38. An individual comes into your office complaining of recent job stress. He notes a chronic pattern of depressed mood, low energy, and eating and sleeping difficulties. Your best initial intervention is to:
A begin individual treatment.
B refer to an expert in vocational issues.
C refer for a psychiatric evaluation.
D explore the possibility of a job change.
ANSWER "C"
Whenever you see any symptoms that might have a medical or psychiatric basis, it's always good to recommend a physical examination. The closest alternative here was to refer to a psychiatrist. Maybe medication is indicated to address the eating and sleeping difficulties and what you suspect might be an underlying depression.
Developmental Psychology
39. Relationships between pre-adolescent siblings are most commonly characterized by
A rivalry and discord.
B conflict and closeness.
C closeness and reciprocity.
D detachment and disinterest.
ANSWER "B"
Sibling relationships in middle childhood are characterized by both conflict and closeness. Sibling rivalry is common during these years and is usually exacerbated by comparisons made between the siblings by parents and other adults. At the same time, pre-adolescent siblings typically serve as a source of companionship, assistance, and comfort for each other during difficult times.
Social Psychology
40. A "script" in social psychology is:
A any verbal communication used to express emotions
B the primary cause of the actor-observer effect
C a set of cognitive distortions
D a representation of a stereotyped sequence of actions
ANSWER "D"
Scripts are schemas that tell us what sequence of actions to expect in certain settings. For example, when you go to a restaurant, your restaurant script causes you to expect that someone will greet you, you will sit down at a table, a server will take your order, etc., in that particular order. You would not expect them to give you the check as soon as you sat down, or to ask you to go to the kitchen to cook your own meal.
Clinical Psychology
41. A difference between the brief dynamic therapies and crisis intervention is:
A brief dynamic therapies focuses on specific symptoms
B crisis intervention focuses on specific symptoms
C crisis intervention views the patient’s symptoms as pathological
D brief dynamic therapies views the patient’s symptoms as pathological
ANSWER "D"
Brief dynamic therapies emphasize specific symptoms (response “A”), viewed as pathological, and addresses underlying personality issues with emphasis on altering the normal level of functioning. Crisis intervention also focuses on specific symptoms (response “B”); however, crisis intervention views symptoms as expected responses to acute trauma and seeks to return the person to the pre-trauma level of functioning.
Diagnosis and Psychopathology
42. Masters and Johnson found that their version of sex therapy, which incorporates education about sexuality, training in communication skills, and the technique known as sensate focus, is most effective for treating:
A premature ejaculation.
B impotence.
C sexual aversion.
D orgasmic disorder.
ANSWER "A"
Masters and Johnson found that close to 100% of individuals with premature ejaculation were helped by their program which incorporated education about sexuality, improving communication, and sensate focus.
Diagnosis and Psychopathology
43. Relapse by smokers who have stopped smoking is most likely to be triggered by the presence of smoking cues in one's surroundings or
A positive affect.
B negative affect.
C inactivity.
D coffee consumption.
ANSWER "B"
According to S. Shiffman et al., the most common precipitants of relapse are the presence of smoking cues and negative affect (First lapses to smoking: Within-subjects analysis of real-time reports, Journal of Consulting and Clinical Psychology, 64(2), 366-389).
Clinical Psychology
44. In their discussion of mental health services for Latino clients, Rogler et al. (1987) describe three ways to increase the cultural sensitivity of therapy for members of this group. These methods include all of the following except:
A incorporating elements of Latino culture into therapy.
B increasing the accessibility of mental health services for Latino clients.
C selecting standard treatments that best fit Latino culture.
D obtaining cultural sensitivity training.
ANSWER "D"
Although all of the actions described in the responses could be called "culturally sensitive," only the first three were described by Rogler and his colleagues, which makes answer D the correct response. [L. H. Rogler et al., What do culturally sensitive mental health services mean? The case of Hispanics. American Psychologist, 1987, 42(6), 656-570.] .
Neuropsychology
45. The Acute phase of Antidepressant treatment is generally
A 4 to 6 weeks
B 4-9 months
C 6-12 months
D 2 years to lifetime
ANSWER "A"
Barry Pierce, M.D. considers 4-6 weeks the Acute phase for antidepressant treatment. The Continuation phase which is used to prevent a relapse is 4-9 months and the Maintenance phase to prevent recurrent episodes is 2 years to lifetime. (Barry Pierce M.D. "Psychopharmacology from A to Z", r cassidy seminars, Santa Rosa, California 1999.).
Ethics and Professional Practice
46. Which of the following statements regarding informed consent procedures in research is least true?
A Psychologists should inform research participants, in language that is reasonably understandable, about the nature of the research.
B Research participants should be informed of significant factors that may affect their willingness to participate in the research.
C Informed consent to research must be appropriately documented.
D Informed consent to research must be obtained in all cases.
ANSWER "D"
In most cases, informed consent to research must be obtained from participants. However, there are some exceptions, such as research involving anonymous questionnaires or naturalistic observations. Ethical standard 8.05 discusses the circumstances under which research does not require informed consent.
Clinical Psychology
47. A social-role theorist might contend that minority clients drop out of treatment more readily when matched with a white therapist, because:
A the therapist indicates discomfort
B higher power or status groups are more adept at reading or perceiving members of lower status groups than those from lower status groups reading people from higher status groups.
C lower power or status groups are more adept at reading or perceiving members of higher status groups than those from higher status groups reading people from lower status groups.
D the client lacks awareness of the differences in power and status.
ANSWER "C"
C. Social-Role theorists assert that individuals in positions of lower power and status are better at reading/perceiving members of higher status groups than are those from higher status groups at reading people from lower status groups. Thus, for example, women may be more adept at attending to men, than men are at attending to the feelings and experiences of women. Similarly white therapists may be less familiar with the culture and life experience of minority clients than are minority therapists with white clients (See: Sue, S. & Zane, N. (1987). The role of culture and cultural techniques in psychotherapy: A critique and reformulation. American Psychologist, 42, 37-45.) .
Clinical Psychology
48. In brief psychodynamically-oriented therapy:
A positive transferences are encouraged and are viewed as essential to treatment progress.
B positive and negative transferences are encouraged early in treatment to ensure that a transference neurosis develops.
C positive and negative transferences are both discouraged because of the here-and-now focus of the treatment.
D negative transferences are encouraged because they are more likely than positive transferences to produce useful information for interpretation.
ANSWER "A"
Brief and long-term psychodynamic psychotherapy share a number of characteristics. A primary difference is that, in brief psychotherapy, a full-scale transference neurosis is discouraged. However, positive transference is seen as an important contributor to therapy progress.
Developmental Psychology
49. An infant is least likely to experience emotional distress due to being separated from his or her parents if the infant is between the ages of:
A 0-8 months.
B 8-12 months.
C 12-16 months.
D 16-24 months.
ANSWER "A"
Separation anxiety typically has its onset at 8 months of age, rises dramatically until the age of 18 months, and then gradually falls off until it becomes negligible between the ages of 24-36 months. Babies between the ages of 0-8 months do not typically protest when separated from their primary caretakers, even though a number of attachment behaviors develop during this time.
Research Design and Statistics
50. When random assignment of subjects to groups is not possible, researchers use:
A Experimental design
B Quasi-experimental design
C Developmental research
D Longitudinal research
ANSWER "B"
Quasi-experimental designs are used when random assignment of subjects to groups is not possible. In true experimental design the investigator randomly assigns subjects to different groups which receive different levels of the manipulated variable. Developmental research involves assessing variables as a function of time. A type of developmental research is longitudinal–in these studies the same people are studied over a long period of time.
Diagnosis and Psychopathology
51. Which of the following statements is true about children who have been diagnosed with a learning disability?
A They usually are below average in general intelligence.
B They usually come from families high in expressed emotion.
C By definition, they cannot be mentally retarded.
D A child who scores one standard deviation above the mean on a test of Reading Achievement and three standard deviations above the mean on a test of general intelligence probably would not be diagnosed with a Learning Disorder.
ANSWER "D"
Reading Disorders, like most mental disorders in the DSM-IV, are only diagnosed when the symptoms cause significant distress or functional impairment. A child whose score on a reading achievement test is one standard deviation above the mean probably would not be impaired or distressed in relation to reading. This is true even though the DSM-IV states that the discrepancy between achievement and intelligence that defines a Learning Disorder is "usually defined as a discrepancy of more than two standard deviations between achievement and intelligence." Here, the word "usually" allows for some exceptions, and, as noted, a child with a clearly above average reading achievement test score would not be functionally impaired in reading.
Industrial/Organizational Psychology
52. According to Holland's personality typology, a person's personality type indicates the:
A person's unconscious occupational abilities
B one occupation the person is capable of performing
C occupational environment in which the person would be most satisfied and productive
D person's occupational interests, but does not predict job satisfaction
ANSWER "C"
Holland classified six personality types and occupational environments, and proposed that the closer the match, or "congruence" between them, the more likely the person will be satisfied, productive, and will stay in that environment. Contrary to A, Holland's personality types are based on self-perceived (not unconscious) abilities and interests. B is incorrect because Holland's typology indicates more than one occupation the person might be capable of performing.
Clinical Psychology
53. A therapist working from the perspective of Minuchin's school of thought would examine a family system in terms of
A multigenerational transmission processes.
B subsystems and boundaries.
C fusion and differentiation.
D communication style.
ANSWER "B"
Salvador Minuchin's Structural Family Therapy is based on and extends general family systems theory. The goal is to restructure maladaptive family structures, including family subsystems and boundaries.
School Psychology/Intelligence
54. In addition to the Vocabulary subtest, which of the following Wechsler subtests are least susceptible to brain damage and, therefore, useful for assessing premorbid intelligence?
A Similarities and Picture Completion
B Information and Picture Completion
C Block Design and Object Assembly
D Information and Arithmetic
ANSWER "B"
Although scores on the Vocabulary, Information, and Picture Completion subtests can be suppressed by certain types of brain injury, they are considered to be most resistant to brain injury and, therefore, the best indicators of premorbid intelligence.
Clinical Psychology
55. Meyer’s (2003) minority stress model identifies distal and proximal factors that contribute to mental health outcomes in gay, lesbian, and bisexual populations. A proximal stressor from the model is:
A homonegativism
B internalized homophobia
C minority group status
D discrimination and violence
ANSWER "B"
B. Meyer’s minority stress model distinguishes between distal factors, which are external, objective events and conditions, and proximal factors, which are an individual’s perceptions and appraisals of events and conditions. The minority stress model identifies three proximal stressors –– expectations of rejection, concealment of sexual orientation, and internalized homophobia (b.). Minority group status (c.) and exposure to “prejudice events” such as discrimination and violence (d.) are identified as a distal events or as an “environmental circumstance” in Meyer’s model. Prejudice, discrimination (d.) and stigma were linked with mental health problems. Homonegativism (a.) refers to the beliefs and values of prejudiced individuals and is not part of the minority stress model. [Meyer, I.H. (2003) Prejudice, social stress, and mental health in lesbian, gay, and bisexual populations: Conceptual issues and research evidence, Psychological Bulletin, 129(5), 674-697.] .
Clinical Psychology
56. According to Janet Helms, a White member of a city council meeting, which consists of members from different racial and cultural backgrounds, is most likely to work cooperatively if the member is in which stage of development:
A contact
B integration
C reintegration
D autonomy
ANSWER "D"
Janet Helms developed the White Racial Identity Development Model, which consists of six stages. "Contact"(A), the first stage, is characterized by ignorance and disregard of any racial differences. The next stage is "disintegration" which involves awareness of racial inequalities which results in moral confusion and conflict. This is followed by "reintegration" (C), in which Whites are viewed as superior to minorities. "Pseudo-Independence" is marked by dissatisfaction with reintegration. "Immersion-Emersion" follows, during which, people embrace their whiteness without rejecting minorities. "Autonomy", the last stage, is reached when a person internalizes a nonracist White identity, whereby similarities and differences are acknowledged but are not perceived as threatening. Note that "Integration" (B) is not one of the stages in Helms' model; rather, it refers to Berry's Acculturation Model, in which a minority has a high retention for the minority culture and high maintenance of the mainstream culture.
Diagnosis and Psychopathology
57. For a diagnosis of Tourette's disorder, a client must have
A multiple vocal tics and at least one motor tic.
B multiple motor tics and at least one vocal tic.
C multiple motor and/or vocal tics.
D multiple motor and vocal tics.
ANSWER "B"
This is a straightforward question requiring you to be familiar with the DSM-IV diagnostic criteria for Tourette's Disorder. For this diagnosis, DSM requires the presence of multiple motor tics and one or more vocal tics for at least one year.
Neuropsychology
58. Which of the following is a non-stimulant medication that alleviates inattention and hyperactivity/impulsivity symptoms in AD/HD?
A methylphenidate
B atomoxetine
C pemoline
D dextroamphetamine
ANSWER "B"
B. In November 2002, the Food and Drug Administration (FDA) approved a new medication called atomoxetine (Strattera) specifically for AD/HD. This medication is neither a stimulant nor an antidepressant. It alleviates inattention and hyperactivity/impulsivity symptoms of AD/HD by affecting specific aspects of the norepinephrine system. This medication is a reuptake inhibitor that acts on the neurotransmitter norepinephrine (which affects blood pressure and blood flow) in the same way that antidepressants act on the neurotransmitter seratonin, allowing the natural chemical to remain longer in the brain before being drawn back up. Because it is a non-stimulant, it may be less objectionable to some families. Nevertheless, it has similar side effects as other medications used for AD/HD. It is a prescription medication, but it is not a controlled substance like a stimulant. This allows medical professionals to give samples and to place refills on the prescriptions. It does not start working as quickly as the stimulants do. Reports suggest that the full effects are often not seen until the person has been taking atomoxetine regularly for 3 or 4 weeks. Methylphenidate (a.), i.e., Ritalin, Concerta, Metadate, pemoline (c.), i.e., Cylert (less commonly prescribed because it can cause liver damage), and dextroamphetamine (d.), i.e., Dexedrine, Dextrostat, along with amphetamines, i.e., Adderall are four main types of stimulants used for the treatment of AD/HD.
Learning Theory and Behavior Therapy
59. The first step a parent takes in teaching a child to feed the family cat is showing her how to put the cat food into the dish. After mastering that task, she is taught to open a can of cat food and then put it into the dish. The child is then taught to open the pantry door, take out a can of cat food, open it, and put it in the dish. This is an example of:
A stimulus control training
B forward chaining
C backward chaining
D sequential training
ANSWER "C"
C. The child has learned a complex behavior by learning the individual responses in the “behavior chain.” The procedure in this question is best described as backward chaining: the parent began by teaching the last behavior and then worked backward from there.
Ethics and Professional Practice
60. A psychologist receives a request for information about treatment for a patient from the patient's insurance carrier. The request for a treatment report includes the appropriate signed authorization from the patient. The psychologist should:
A not provide the information because this is still an unsettled issue.
B provide only the dates of service and the diagnosis.
C refuse the request unless the patient himself or herself asks that it be sent.
D provide the information requested.
ANSWER "D"
This ethics question will have some of you stumped unless you deal with these types of clinical situations regularly. In practice, clinicians often get requests for information, especially with insurance companies using managed-care organizations to approve visits. Similar authorizations are necessary when information is released from a hospital, from a clinic, and so forth. The confidentiality of information belongs to the patient. It is up to him or her to waive that confidentiality, as we assume he or she has done by signing the release. In addition, such authorizations should specify the type and extent of information that the patient authorizes to be released, for what purposes, and to whom.
Diagnosis and Psychopathology
61. Which of the following is the most common functional disorder in the elderly?
A Delusional Disorder
B Depressive Disorder
C Dementia
D Bipolar Disorder
ANSWER "B"
To answer this question, it helps to know what the term "functional disorder" means. It is actually a somewhat outdated term: the DSM-III-R distinguished organic mental disorders, which are due to a known physiological cause, from functional disorders, which may or may not have a physiological component but are not directly caused by a known physiological factor. The DSM-IV divides what were previously called organic disorders into two categories: Mental Disorders Due to a General Medical Condition and Substance-Induced Disorders; disorders that don't fit into either of these categories are called primary mental disorders. Anyhow, this question illustrates that you sometimes have to be familiar with terms that are no longer part of the "official" lingo. If you knew the meaning of the term functional disorder, you would have eliminated Dementia as a choice, since Dementia is always either due to the effects of a general medical condition or is substance induced (or, using the old language, is always an organic mental disorder). Of the remaining choices, the prevalence of Depressive Disorders in the elderly is 5-10%, the prevalence of Delusional Disorder is approximately 4%, and the prevalence of Bipolar Disorder is 1%. So "B" is the correct answer.
Neuropsychology
62. Damage to the orbitofrontal cortex is most likely to result in:
A impaired depth perception
B impaired motor coordination
C altered emotional behaviors
D left-right confusion
ANSWER "C"
The orbitofrontal cortex is part of the prefrontal cortex located just above the eye sockets. Damage to the orbitofrontal cortex is most likely to alter emotional behavior. The orbitofrontal cortex is believed to play a role in excitability, behavioral inhibition, personality, and judgment.
The famous case of Phineas Gage, the 19th century railroad worker who survived an iron rod shooting through his head, suffered damage to his orbitofrontal cortex which is believed to have caused changes in his emotions and behaviors.
Diagnosis and Psychopathology
63. A client who claims that the disk jockey on a local rock n'roll radio station is speaking directly to him most likely:
A is experiencing a systematized delusion.
B is experiencing a delusion of reference.
C is experiencing a persecutory delusion.
D has heard the same song over and over again on the local "classic rock" station one too many times.
ANSWER "B"
A delusion of reference is one in which the person believes that objects or events in the immediate environment have an unusual and particular significance to him or her. In a persecutory delusion, the person feels that he or she is being mistreated in some way or conspired against; this is not the case here. Systematized delusions are organized around a coherent theme; there is no evidence that this is the case. Hearing the same song over and over again can drive one crazy, but probably not to the extent described in the question.
School Psychology/Intelligence
64. For the WISC-III, which of the following would have the lowest reliability?
A Verbal IQ score
B Performance IQ score
C subtest scores
D Verbal-Performance difference score
ANSWER "D"
If you take a difference between two scores, you are, obviously, using two measures. Since each is less than 100% reliable, you are going to compound the total error variance (i.e., the total unreliability). The error contained in both tests would exist in the difference score. Thus, using two measures which are less than perfectly reliable, you will end up with a score which is less reliable than either of the two you started with.
Diagnosis and Psychopathology
65. During an evening interview, a client displays incoherence, disorientation, distractibility, fragmented delusions, and impaired recent memory. When the client is seen the next morning, his symptoms have remitted. The MOST likely diagnosis for this patient is:
A amnestic disorder
B delirium
C brief psychotic disorder
D substance intoxication
ANSWER "B"
Even though there is no evidence of an underlying medical condition or substance use, delirium is the best response based on the information about the symptoms and their duration. Delirium involves a disturbance in consciousness with a change in cognition or the development of perceptual abnormalities. An individual exhibits a reduced awareness of his environment, shifts in attention, and distractibility, and changes in cognition can include memory loss, disorientation to time and place, and impaired language. Associated perceptual abnormalities can include illusions, hallucinations, or other misperceptions. As in this case, the symptoms of delirium usually fluctuate in the course of a day, and depending on the cause of the disorder, may remit within a few hours or persist for weeks. Response “D” may have been difficult to rule out as the symptoms could represent a substance-induced delirium; however the symptoms appear to be in excess of symptoms usually associated with “substance intoxication” or an intoxication syndrome. In contrast to brief psychotic disorder (response “C”), this client’s psychotic symptoms fluctuate, are fragmented, and occur in the context of reduced awareness of the environment, shifts in attention, and distractibility which are associated with delirium. An amnestic disorder (response “A”) is not diagnosed if memory difficulties occur in the course of delirium, or as with this client, the memory impairment is accompanied by reduced consciousness or other cognitive deficits.
Social Psychology
66. A man who is very shy goes out with a friend. Suddenly, a crowd breaks out into a riot and the man becomes very aggressive. This is an example of
A deindividuation.
B social power.
C social comparison.
D reactance.
ANSWER "A"
Deindividuation, which occurs under conditions of anonymity, refers to the loss of a person's identity and a loosening of normal restraints against deviant behavior. It has been offered as an explanation for mob violence, and it describes what is happening in the question.
Social power (B) refers to the means by which a person is influenced by another (i.e., expert, legitimate, reward, coercive, or referent power). Here, the person is being influenced by a crowd rather than another individual, so deindividuation is a more accurate term. Social comparison (C) refers to the process by which we evaluate our opinions and abilities by comparing them to those of others. Reactance (D) refers to the theory that a person will react against perceived threats to freedom by choosing a course of action that is the opposite of what he or she is being pressured to do.
Clinical Psychology
67. Which of the following statements is true regarding electromyograph (EMG) biofeedback?
A It is more effective in treating tension headaches than migraine headaches.
B It is more effective in treating migraine headaches than tension headaches.
C It is equally effective in treating tension headaches and migraine headaches.
D It is ineffective in treating tension and migraine headaches.
ANSWER "A"
Overall, biofeedback works equally well for tension and migraine headaches. However, a different modality of biofeedback is used for each of these conditions. EMG biofeedback, in which the person is trained to decrease muscle tension of the facial and/or neck muscles, is the most commonly used modality for biofeedback treatment of tension headaches; this is why A is the best answer. For migraine headaches, thermal hand warming biofeedback, whereby the person is trained to warm his or her hands, is the most commonly used modality.
Research Design and Statistics
68. A psychologist is conducting research to evaluate the effectiveness of three predictor tests of overall mental health he has developed. He administers the predictors to 35 individuals randomly chosen from the population of interest and obtains a squared multiple correlation coefficient (R2) of .47. If the psychologist administers the predictors to another 70 individuals drawn from the same population, the best prediction is that, he would obtain an R2 that is:
A lower than .47.
B about equal to .47.
C slightly to moderately higher than .47.
D much higher than .47.
ANSWER "C"
The principle behind this question is that the greater the range of scores in both the predictor(s) and the criterion, the higher the validity coefficient will be. If you administer the predictors to 70 people as opposed to 35, you are likely to get a somewhat greater range of scores in the former case. Therefore, you will get a somewhat higher correlation coefficient. Choice D, a much higher correlation coefficient is not a good answer. Increasing the range of scores can only do so much for your correlation coefficient, especially if you already have a reasonably representative sample to begin with. Increasing the sample size from 35 to 70, for example, will not turn a poor set of predictors into a good one.
Some of you might have gone for choice A, thinking that, due to shrinkage, the correlation coefficient would be smaller. Shrinkage, however, is associated with the development of a predictor or set of predictors. It occurs when, based on research with one sample, items for a predictor are chosen from a larger pool, and the newly developed predictor is then tested on a second sample. The correlation coefficient for the second sample is likely to be smaller, because the predictor was "tailor made" for the first sample. In this question, however, the predictors are not in the process of development, and the first group of 35 people is not a validation sample (i.e., a sample of people used to determine which items to retain for the final version of the test). To see if you understand this distinction, try to rewrite the question, changing as few words as possible, so that A becomes the best answer.
School Psychology/Intelligence
69. When using the WISC-III, the Symbol Search and Coding subtests provide information on
A learning disabilities.
B brain dysfunction.
C perceptual organization.
D processing speed.
ANSWER "D"
Processing speed is one of four factors that can be assessed with the WISC-III and is a measure of mental and motor speed while solving nonverbal problems. (The other factors are verbal comprehension, perceptual organization, and freedom from distractibility).
Diagnosis and Psychopathology
70. For women in general, the typical pattern of depression and dysfunctional physical symptoms accompanying a menstrual cycle is that:
A there is no typical pattern.
B the symptoms occur shortly before and terminate shortly after the onset of menses.
C the symptoms occur shortly after the onset of menses.
D the symptoms occur and terminate shortly before the onset of menses.
ANSWER "A"
The key words in this question were "for women in general." You had to know that PMS is actually not typical of most women. While it is not uncommon for women to have mild psychological symptoms during the latter part of the luteal phase, there is no specific pattern typical of most women. Make sure you read each word in the question. You might want to use your pencil and point to each word as you read. This reduces the chance of missing something. If the question were about women who do suffer from PMS, the answer would be B.
Social Psychology
71. A high achiever would most likely attribute failures to
A internal, unstable, and controllable factors.
B internal, stable, and uncontrollable factors.
C external, stable, and controllable factors.
D external, unstable, and uncontrollable factors.
ANSWER "A"
Research by Weiner and others has suggested that people who have high expectations for future performance (such as high achievers) tend to attribute their failure to a lack of effort. Effort is an internal, unstable, and controllable factor. Most research suggests that of these dimensions, stability is the most important in expectations for future achievement. The idea is that, if you attribute failure to an unstable cause, you must expect that you will not fail in most situations.
Diagnosis and Psychopathology
72. Hallucinations experienced by Schizophrenics are primarily:
A visual.
B auditory.
C tactile.
D somatic.
ANSWER "B"
Hearing voices is a modal symptom of this disorder.
Developmental Psychology
73. A college student tells you she has just read that adolescent females are faced with a conflict between their own strengths and accomplishments and the stereotypic feminine roles they are expected to adopt. Apparently, this student has just read the work of:
A Kohlberg.
B Erikson.
C Gilligan.
D Ainsworth.
ANSWER "C"
You may have been able to answer this question through the process of elimination. Kohlberg, Erikson, and Ainsworth didn't distinguish between girls and boys in their theories. That leaves Gilligan who is best known for her work on moral development in girls but has also addressed other developmental issues.
Research Design and Statistics
74. In the analysis of the effects of two independent variables, multiple regression analysis is sometimes used as a substitute for the factorial ANOVA. One advantage of using multiple regression as opposed to a factorial ANOVA is that:
A multiple regression analysis can be used for multiple dependent variables as well as multiple independent variables.
B continuous or categorical data (as opposed to solely categorical data) can be used to measure the independent variables in multiple regression analysis.
C the use of multiple regression allows one to estimate the probability that obtained differences on the dependent variable between groups represent true population differences.
D when multiple regression is used and a significant result is obtained, the conclusion that there is a causal relationship between the independent variables and the dependent variables is more plausible.
ANSWER "B"
One limitation of the ANOVA technique is that independent variables must be divided into categories for the analysis to be conducted. In multiple regression, the researcher has the choice of using categories or continuous data (e.g., scores on a test) to measure the independent variables. This is considered an advantage of regression, because it allows for the data to provide more precise and specific information about the variables being measured.
Choice A is not true of multiple regression; it is designed for use with one dependent variable only. Choice D is also not true; the strength of the conclusion that variables are causally related depends on the research design, not the statistical analysis. And choice C is true of both multiple regression and ANOVA, since they are both inferential statistical methods.
Learning Theory and Behavior Therapy
75. The notion that reinforcement is not necessary for learning and that learning does not always manifest in performance is referred to as:
A latent learning
B place learning
C observational learning
D insight learning
ANSWER "A"
A. According to Tolman’s Cognitive Learning Theory, learning is the result of conditioning and cognitive understanding. The acquisition of cognitive structures or cognitive maps underlies the concept of latent learning or learning that occurs without reinforcement and does not immediately manifest in behavior. Place learning (b.) refers to Tolman’s description of learning places or paths instead of a series of movements in response to specific stimuli. Observational learning (c.) refers to learning through watching a model behave and then imitating the modeled behavior. Insight learning (d.) is an “a-ha!” experience or a sudden novel solution, which Kohler saw as the result of a sudden internal cognitive restructuring of the environment.
Neuropsychology
76. In most species of animals the differences between males and females in body size and shape is referred to as:
A androgyny
B sexual bifurcation
C gender dichotomy
D sexual dimorphism
ANSWER "D"
The term "sexual dimorphism" may be new to you, but now you know that it refers to any consistent differences between males and females in size or shape. Sexual dimorphism enables animals to readily identify males and females of their species which serves to facilitate mating.
Clinical Psychology
77. Beck identified a number of cognitive distortions including the tendency to focus solely on a detail that is taken out of context. He termed this:
A Selective abstraction
B Overgeneralization
C Personalization
D Dichotomous thinking
ANSWER "A"
Focusing solely on a detail is an example of selective abstraction. Overgeneralization is abstracting a general rule from one or two situations and then broadly applying it to other situations. Personalization is attributing external events to oneself without evidence of a causal correlation. Dichotomous thinking is categorizing experiences in one of two extremes.
Developmental Psychology
78. Which of the following best reflects the research findings on stepparenting?
A During late childhood and early adolescence males respond more positively to their stepfathers than do females.
B During late childhood and early adolescence females respond more positively to their stepfathers than do males.
C Stepfathers have more frequent contact with their stepchildren than do stepmothers.
D Stepparents most often utilize an authoritative parenting style.
ANSWER "A"
Most research has found that girls have more difficulty adjusting to their mother's remarriage than do boys. This is particularly the case during childhood and early adolescence. Older adolescent males, on the other hand, often continue to have problems with the adjustment. The reverse of choice C is true; that is, stepmothers typically have more frequent contact with their stepchildren than do stepfathers, although these interactions are often abrasive. And in contrast to choice D, stepparents have been found to rely heavily on authoritarian (rather than authoritative) parenting.
Social Psychology
79. Cognitive dissonance theory predicts that, when a college student is not accepted by the sorority she most wanted to join, the student will
A blame the sorority for not being accepted.
B blame herself for not being accepted.
C decide that she doesn't really want to be in the sorority after all.
D feel depressed until an alternative sorority accepts her.
ANSWER "C"
Cognitive dissonance theory predicts that, when we have two conflicting cognitions (e.g., I like the club but they don't want me"), we'll be motivated to reduce the tension that this causes by changing one of our cognitions. Since the student can't change the sorority's decision in this situation, she's likely to change her attitude toward the sorority.
Test Construction
80. If you find that your job selection measure yields too many "false positives," what could you do to correct the problem?
A raise the predictor cutoff score and/or lower the criterion cutoff score
B raise the predictor cutoff score and/or raise the criterion cutoff score
C lower the predictor cutoff score and/or raise the criterion cutoff score
D lower the predictor cutoff score and/or lower the criterion cutoff score
ANSWER "A"
On a job selection test, a "false positive" is someone who is identified by the test as successful but who does not turn out to be successful, as measured by a performance criterion. If you raise the selection test cutoff score, you will reduce false positives, since, by making it harder to "pass" the test, you will be ensuring that the people who do pass are more qualified and therefore more likely to be successful. By lowering the criterion score, what you are in effect doing is making your definition of success more lax. It therefore becomes easier to be considered successful, and many of the people who were false positives will now be considered true positives.
If you understand concepts in pictures better than in words, refer to the Test Construction section, where a graph is used to explain this idea.
Industrial/Organizational Psychology
81. A reason why the rational-economic model of decision-making usually does not prove viable in an organization is that
A it allows for the consideration of a limited number of options.
B it places too many demands on the individual and the organization.
C it arouses resentment in organization members who are excluded from the decision-making process.
D humans are irrational and emotional by their nature.
ANSWER "B"
The rational-economic model assumes that decision-making involves a rational process in which all alternatives are considered and the best possible decision is eventually made. The model requires that decision-makers obtain information about and consider all possible alternatives before making a decision. This requirement places an inordinate demand on the people involved. Thus, in the real world, decisions are usually made based on the information that is available given restraints of resources such as time, money, and personnel.
Diagnosis and Psychopathology
82. Medical treatment regimens for chronic illness most often results in:
A greater compliance for children as compared to adolescents
B greater compliance for adolescents as compared to children
C equal compliance for children and adolescents
D greater compliance for girls and adolescent females as compared to boys and adolescent males
ANSWER "A"
Compliance with medical treatment regimens, such as those designed to manage diabetes, tends to be lower for adolescents as compared to children or adults. There are many reasons for this, including adolescents' greater desire to be similar to their peers and independent from the restrictions of their parents.
Ethics and Professional Practice
83. Dr. Fastbuck has been hired by a for-profit mental health hospital corporation as a supervisor of two psychological interns, Dr. Newbie and Dr. Greenhorn. The hospital has promised both interns that they would receive training in psychological assessment. The agency needs at least one of these interns trained quickly, due to a backlog of work on financially lucrative psychological assessments. Dr. Fastbuck decides that Dr. Newbie will receive extensive experience and training in testing, while Dr. Greenhorn will get very little training, because training one of them can be done more quickly than training both. Dr. Fastbuck has acted:
A ethically, because finishing the assessments quickly is in the best interests of clients.
B ethically, because, in light of the situation, Dr. Fastbuck does not have any alternative.
C unethically, because decisions such as these cannot be based on financial concerns.
D unethically, because Dr. Fastbuck is failing to meet his responsibility as a supervisor.
ANSWER "D"
Ethical Standard 2.05 applies to this situation. It states in part that “Psychologists who delegate work to employees, supervisees, or research or teaching assistants or who use the services of others, such as interpreters, take reasonable steps to (1) avoid delegating such work to persons who have a multiple relationship with those being served that would likely lead to exploitation or loss of objectivity; (2) authorize only those responsibilities that such persons can be expected to perform competently on the basis of their education, training, or experience, either independently or with the level of supervision being provided; and (3) see that such persons perform these services competently." In this situation, in light of what Dr. Greenhorn was hired for, Dr. Fastbuck is not providing proper supervision. It could be argued that the hospital's policy prevented fulfillment of this obligation, but there is no indication in the question that Dr. Fastbuck has attempted to correct the situation.
Diagnosis and Psychopathology
84. A client of yours is insensitive to criticism or approval, doesn't make eye contact during conversations, rarely smiles or frowns, didn't do well in school, and expresses little interest in a sexual relationship or marriage. These symptoms are most suggestive of
A Schizoid Personality Disorder.
B Schizotypal Personality Disorder.
C Avoidant Personality Disorder.
D Asperger's Disorder.
ANSWER "A"
This client is exhibiting detachment from social relationships and a lack of emotional responsivity, which are the core features of Schizoid Personality Disorder. With Schizotypal Personality Disorder (answer B), there would be cognitive and perceptual disturbances and eccentricities in behavior; with Avoidant Personality Disorder, the client would express a desire for relationships but avoid them due to fear of criticism and humiliation. For a diagnosis of Asperger's Disorder (answer D), there would have to be more severe social impairment and the presence of stereotyped behaviors.
Test Construction
85. Discriminant and convergent validity are classified as examples of:
A construct validity.
B content validity
C face validity.
D concurrent validity.
ANSWER "A"
There are many ways to assess the validity of a test. If we correlate our test with another test that is supposed to measure the same thing, we'll expect the two to have a high correlation; if they do, the tests will be said to have convergent validity. If our test has a low correlation with other tests measuring something our test is not supposed to measure, it will be said to have discriminant (or divergent) validity. Convergent and divergent validity are both types of construct validity.
Developmental Psychology
86. Michael Rutter, a key figure in the field of developmental psychopathology, notes that parental divorce does not have the same effects on all children. According to Rutter, such differences are related to:
A the child's cognitive understanding of the causes of the divorce.
B the child's early social interactions, especially interactions with his or her parents.
C the nature of the parents' relationship following the divorce.
D the custodial parent's social support and financial status.
ANSWER "B"
Rutter and others interested in developmental psychopathology have attempted to identify the factors that account for the continuities and discontinuities in child psychopathology. Rutter has focused primarily on variations in social relationships that act as high risk or protective factors.
Industrial/Organizational Psychology
87. Holland's social type is most similar to which other one of his types?
A artistic
B investigative
C realistic
D conventional
ANSWER "A"
Holland's theory categorizes individuals based on their resemblance to six personality types: realistic, investigative, artistic, social, enterprising, and conventional (you can remember the sequence as "RIASEC"). The personality types reflect a person's primary interest, or an environment's primary characteristic; for example, the social type enjoys working with others and avoids ordered, systematic activities that involve tools or machinery. There is overlap between the types, though some types overlap more than others. For example, the social type is relatively more similar to the artistic and enterprising types, and less similar to the realistic, investigative, and conventional types.
Community Psychology
88. Which of the following groups has the highest rate of completed suicide?
A Caucasian females
B Caucasian males
C African-American females
D African-American males
ANSWER "B"
Across all age groups, Caucasian males have a higher rate of completed suicide that the other groups listed by the choices. However, the rate of suicide in African-American males is rising the fastest.
Clinical Psychology
89. A student seeks counseling at the university counseling center due to feelings of failure as a student. When asked about his grades, he states that his grade point average is 3.9 but thinks that he should be doing better. He is most likely using which of the following cognitive distortions?
A arbitrary inference
B overgeneralization
C personalization
D dichotomous thinking
ANSWER "A"
Arbitrary inference occurs when one draws a specific conclusion without supporting evidence, or even in the face of contradictory evidence. This best applies to the student in this question. Despite having a nearly perfect GPA, the student believes he's a failure and should be doing better. Overgeneralization (B) is the application of a general rule based on a few isolated incidents. Personalization (C) is attributing external events to oneself without evidence of a causal connection. And dichotomous thinking (D) is categorizing experiences into one of two extremes.
School Psychology/Intelligence
90. To be consistent with the Federal Educational Rights and Privacy Act (the Buckley Amendment), a school counselor must keep in mind that
A it is necessary to obtain the signed consent of a student's parents before releasing any information from the student's file to officials of another school where the student is planning to enroll.
B student records, including identifying information, must be released to designated federal and state educational authorities if needed in connection with the evaluation of federally-sponsored educational programs.
C parents have the right not only to inspect and review their children's school records but also to challenge the contents of records.
D all of the above.
ANSWER "C"
Only C is a requirement of the Buckley Amendment. Written consent is not needed in the situation described in choice A, as long as the parents have been notified of the transfer of records to the new school. And although the records must be supplied in the situation presented in B, identifying information must be removed from the records.
Neuropsychology
91. The occipital lobe is to the temporal lobe as
A vision is to hearing.
B receptive language is to expressive language.
C vision is to touch.
D hearing is to smell.
ANSWER "A"
The visual cortex, which is responsible for visual perception, recognition, and memory, is located in the occipital lobe of the cerebral cortex. The auditory cortex, which is involved in the mediation of auditory sensation and perception, is located in the temporal lobe.
Research Design and Statistics
92. In designing a research study, you take a number of steps that have the effect of reducing beta. This means that you have reduced the probability of:
A retaining a true null hypothesis.
B retaining a false null hypothesis.
C rejecting a true null hypothesis.
D rejecting a false null hypothesis.
ANSWER "B"
Beta is the probability of making a Type II error, or of retaining a false null hypothesis. In plain language, it is the probability of failing to detect a true effect.
Learning Theory and Behavior Therapy
93. The behavioral technique known as flooding is based on the theoretical principle of
A reciprocal inhibition.
B covert desensitization.
C classical extinction.
D negative reinforcement.
ANSWER "C"
In flooding, the patient is exposed to a feared stimulus. The technique is based on the principle of classical extinction, which involves repeatedly presenting a conditioned stimulus without the unconditioned stimulus. The idea is that the fear developed through classical conditioning, or a pairing of a conditioned stimulus (the feared stimulus) and an unconditioned stimulus (a stimulus that naturally causes fear). For instance, a fear of dogs might have been developed through a pairing of a dog (the conditioned stimulus) and a frightening event associated with a dog, such as a dog bite (the unconditioned stimulus). Classical extinction involves "unpairing" the conditioned and the unconditioned stimulus -- for instance, repeatedly exposing the person to dogs that don't bite.
Industrial/Organizational Psychology
94. In applied industrial settings, Maslow's theory has been shown to be:
A applicable for lower-level employees only.
B appropriate for supervisors but not managers.
C applicable across the board.
D not applicable.
ANSWER "D"
Maslow's theory, as it applies to workplace settings, has not been supported by research. For instance, the notion that lower level needs, such as pay and job security, cease to be important once they are satisfied has not been shown to be true.
Learning Theory and Behavior Therapy
95. A child is reinforced for cleaning up her room and for doing homework. Reinforcement for the homework is stopped. One could predict that cleaning up will:
A increase and doing homework will decrease.
B decrease and doing homework will also decrease.
C increase and doing homework will increase.
D decrease and doing homework will increase.
ANSWER "A"
This question has to do with the behavioral contrast effect. If we are reinforced for performing two different operants, and reinforcement for one of these behaviors stops, we tend to increase the rate of the remaining reinforced behavior. That is probably because the reinforcement that remains seems to become more valuable.
Developmental Psychology
96. Erik Erikson is known for all of the following except:
A eight developmental stages
B ecological model
C identity Crisis
D psychosocial moratorium
ANSWER "B"
Erik Erikson is best known for his psychosocial developmental model which consists of 8 stages spanning from birth to death. Each stage is characterized by a "crisis," the most famous of which, the "identity crisis" ("C"), is experienced in adolescence during the Identity vs. Role Confusion Stage. During this stage many adolescents also take a time-out by withdrawing from their responsibilities and exploring alternative identities, which Erikson and Marcia referred to as a "moratorium" ("D"). In fact, Erikson even encouraged adolescents to take such a moratorium. The only choice offered which is not associated with Erickson is the "ecological model" ("B") which was developed by Bronfenbrenner to describe the four environmental systems which influence development: the microsystem, mesosystem, exosystem, and macrosystem.
Industrial/Organizational Psychology
97. According to Herbert Simon's administrative model, decision-makers in organizational settings typically chose the:
A least expensive choice
B most "satisficing" choice
C best choice after weighing all the alternatives
D worst choice after weighing all the alternatives
ANSWER "B"
Herbert Simon earned a Nobel Prize in economics, but was also known for his theories in psychology, computer science (artificial intelligence), and administration. The one thread through all of his work was his interest in decision-making and problem solving. His "administrative" model maintains that decision-makers cannot always afford to be rational, instead, they must choose the first solution that is minimally acceptable or "satisficing." This is in contrast to another decision making theory, the "rational-economic model" in which decision-makers attempt to find the optimal solution to a problem.
Research Design and Statistics
98. Which of the following is NOT a disadvantage of a repeated measures design?
A multicollinearity
B autocorrelation
C practice effects
D carryover effects
ANSWER "A"
A "repeated measures" design, sometimes referred to as a "within-subjects design," uses more than one measurement of a given variable for each subject. For example, longitudinal studies and pre-test/post-test designs measure the same subjects multiple times. These designs have several disadvantages including: "Autocorrelation" (B), which means that observations obtained close together in time from the same subjects tend to be highly correlated. This violates the independence of observations assumption made by statistical tests. "Practice effects" (C), "carryover effects" (D), and "order effects" all refer to systematic changes in subjects' performance due to prior exposure to a treatment condition or measurement. However, multicollinearity (A) refers to a problem associated with multiple regression which occurs when two or more predictors are highly correlated with each other.
Social Psychology
99. Self-perception theory proposes that people base their:
A beliefs on their feelings
B behaviors on their attitudes
C attitudes on their behaviors
D attitudes on their feelings
ANSWER "C"
Bem's self-perception theory contradicts the more intuitive belief that we make conclusions about ourselves based solely on internal states. Based on the work of B. F. Skinner, Bem's self-perception theory proposes that when internal cues are weak or difficult to interpret, we rely on observations of our behavior and/or circumstances in which this behavior occurs to .
Diagnosis and Psychopathology
100. According to DSM-IV, patients who meet some but not all of the criteria for a particular diagnosis can still be assigned that diagnosis. This reflects the fact that:
A DSM-IV relies on a dimensional rather than a categorical approach to diagnosis.
B DSM-IV relies on a categorical rather than a dimensional approach to diagnosis.
C DSM-IV combines aspects of categorical and dimensional diagnosis.
D DSM-IV's approach to diagnostic classification is not valid.
ANSWER "B"
DSM-IV uses a categorical approach to diagnosis. This means that mental disorders are divided into types based on criteria sets with defining features. An alternative to the categorical approach would be a dimensional approach, which would involve quantifying patients' symptoms based on their severity. Some have suggested that the categorical approach is inappropriate for diagnosis of mental disorders because it misleadingly suggests that the categories of disorders and individuals within the same category are homogeneous in all relevant ways. To provide some degree of compensation for these limitations, DSM-IV diagnoses often include a polythetic criteria set, which means that, for a diagnosis to be made, a person may present with some but not all of the diagnostic criteria. In addition, DSM-IV reminds the clinician to include other information besides diagnoses in evaluation reports.
Research Design and Statistics
101. If you had a categorical variable and a continuous variable which of the following tests would you use?
A Point-biserial
B Eta
C Spearman Rho
D Tetrachoric
ANSWER "A"
The point-biserial correlation is used with a continuous variable and one dichotomous (categorical). Answer "B", Eta, is used when there is a nonlinear relationship between variables. "C", the Spearman Rho is a correlation coefficient used to correlate two variables that have been ordinally ranked. "D", a tetrachoric coefficient, is used when both variables are artiifiicially dichotomized.
Industrial/Organizational Psychology
102. Two women golfers are competing in a golf tournament. Woman A is told by the coach to do her best. Woman B is told by her coach to try to shoot one under par on Holes 1, 4, and 7 and 11, and make par on all the rest. Which golfer is likely to do the best according to Locke?
A Golfer A
B Golfer B
C They'll do the same
D Golfer C, who will not be distracted from the game by verbal directions from a coach
ANSWER "B"
According to Locke's 1970 goal-setting theory, goals serve two purposes–they are a basis for motivation and they direct behavior. Goal attainment is maximized when goals are specific and moderately difficult and when frequent feedback about progress toward goal achievement is provided. Locke is associated with Industrial Organizational Psychology, even though this question seems like it could be from the Social Psychology domain. Questions on the EPPP are often "disguised" in this way.
Developmental Psychology
103. Which of the following is achieved during the operations stages?
A magical thinking
B decentration
C symbolic thought
D object permanence
ANSWER "B"
The "operations stages" refer to Piaget's concrete operations and formal operations stages. The stages prior to the operations stages are the sensorimotor and "preoperations," which, as its name implies, occurs before the development of operations abilities. The development of decentration and reversibility form the basis for conservation, which is the characteristic achievement of concrete operations (7 to 12 years). Object permanence (D) is achieved during the sensorimotor stage (birth to 2 years). Symbolic thought (C) develops during the preoperations stage (2 to 7 years) resulting in the development of language and pretend play. Magical thinking (A) is also characteristic of preoperational thought.
Neuropsychology
104. A potential drawback of the SSRIs is that they
A are cardiotoxic and, consequently, dangerous in overdose.
B initially cause cognitive impairment, especially in older patients.
C may trigger mania.
D may initially worsen sleep and anxiety symptoms.
ANSWER "D"
In general, the SSRIs are safer than the tricyclics. For instance, they are not cardiotoxic and do not cause cognitive impairments. However, they may exacerbate anxiety and sleep symptoms initially.
Diagnosis and Psychopathology
105. A woman with Schizophrenia is most likely to have a _________ with Schizophrenia.
A mother
B father
C monozygotic sister
D dizygotic brother
ANSWER "C"
Biological relatives of individuals with Schizophrenia have an increased risk of developing the disorder. The more similar their genes are, the greater the risk. Monozygotic (identical) twins have the same genes and studies have shown that they have a 46% concordance rate for Schizophrenia. Dizygotic (fraternal) twins only have a 17% concordance rate. The likelihood of one of the parents or a non-twin sibling having or developing Schizophrenia is about 10%.
Research Design and Statistics
106. In forward stepwise multiple regression analysis, the goal is to obtain the smallest subset of predictors to account for the largest amount of variability in the criterion variable. Statistically, this involves:
A adding predictors to the multiple regression equation and determining, through statistical analysis, if the coefficient of multiple determination is significantly increased
B using the correction for attenuation formula to estimate what the predictive power of the multiple regression equation would be if all the predictors had perfect reliability
C using the Spearman-Brown Prophecy formula to estimate the magnitude of the multiple correlation coefficient if all the predictors were used, and comparing the result to the magnitude of the coefficient when different subsets of the predictors are used
D administering different subsets of the predictors to two validation samples, and conducting statistical analyses to estimate the degree of shrinkage in the multiple correlation coefficient from the first to the second validation sample
ANSWER "A"
The goal of stepwise regression analysis is to derive the smallest subset of predictors, out of a larger set, that maximizes the ability to predict outcome on a criterion variable. There are two types of stepwise multiple regression: forward and backward. In forward stepwise regression, predictors are successively added to the multiple regression equation. With each addition, an analysis is conducted to determine if the predictive power of the equation is increased. Predictive power is measured by the squared multiple correlation coefficient (also known as the coefficient of multiple determination). Thus, choice A is the best answer.
Diagnosis and Psychopathology
107. Which of the following is true regarding Schizophrenia Type I and Schizophrenia Type II?
A Traditional antipsychotic medications are less effective for Type II than for Type I.
B Patients with Type II have more "positive" symptoms compared to patients with Type I.
C Patients with Type II are more distractable compared to patients with Type I.
D Patients with Type II have a better prognosis compared to patients with Type I.
ANSWER "A"
Crow (1980) classified Schizophrenia as two distinct syndromes: Type I and Type II. Type I is characterized by positive symptoms (e.g., hallucinations, delusions) and over-responsiveness/distractibility. Type I also responds well to traditional neuroleptics and has a good chance of partial or complete recovery. Type II is characterized by the opposite: negative symptoms (e.g., flat affect, anhedonia) and under-responsiveness/slowness to respond, a poor response to traditional neuroleptics, and poor chance of recovery (T. J. Crow, Molecular pathology of schizophrenia: More than one disease process? British Medical Journal, 1980, 280, 66-68).
Industrial/Organizational Psychology
108. Of the following, the best predictor of job success would likely be a
A general cognitive ability test.
B personality test.
C structured interview.
D vocational interest inventory.
ANSWER "A"
Research on the validity of selection measures generally finds the cognitive ability tests and biodata are 1-2 in terms of predictive validity. Although validity coefficients will vary from study to study, the most widely accepted conclusion is that general ability tests are the best overall predictors.
Research Design and Statistics
109. The upper and lower limits of the standard error of measurement for a test with a mean of 80 and standard deviation of 10 are:
A 0 to 80
B 0 to 10
C -1.0 to +1.00
D 0 to +1.0
ANSWER "B"
B. There is no error in measurement and the standard error of measurement equals zero when the reliability coefficient of a test is equal to +1.0 (the highest reliability coefficient possible). The standard error of measurement equals the standard deviation of the test scores when the test’s reliability coefficient is equal to 0 (the lowest possible). It is helpful to know the formula for the standard error of measurement: the standard error of measurement equals the standard deviation times the square root of one minus the reliability coefficient, when answering this type of question.
Learning Theory and Behavior Therapy
110. Which of the following would most likely be remembered?
A a political advertisement expressing views consistent with the viewer's beliefs
B a political advertisement expressing views opposite to the viewer's beliefs
C an emotionally-charged event
D an emotionally-charged event which was expressively suppressed
ANSWER "C"
As you may have guessed, memory is enhanced for emotionally-charged events. Research has also found that this enhancement is not solely due to increased attention (S. Christianson, E. Loftus, H. Hoffman, & G. Loftus. Eye Fixations and Memory for Emotional Events, Journal of Experimental Psychology: Learning, Memory, and Cognition, 1991, 17(4), 693-701). Attempts to suppress emotion (D) have been found to decrease one's memory for the event. Examples of emotional suppression include biting one's lip to keep from crying or maintaining a poker face after being dealt a great hand of cards (J. Richards & J. Gross. Emotion regulation and memory, Journal of Personality and Social Psychology, 2000, 79(3), 410-424).
Clinical Psychology
111. Hypnosis would least likely be used in the treatment of:
A Substance Abuse.
B Obsessive-Compulsive Disorder.
C Specific Phobia.
D chronic pain.
ANSWER "B"
Hypnosis is contraindicated in individuals who have difficulty giving up control, such as obsessive-compulsive patients, or those who have difficulty with basic trust, such as paranoid patients. By contrast, hypnosis is often employed in the treatment of the other disorders. In the treatment of Substance Abuse, varying degrees of success have been reported; however, it continues to be used. It is also commonly used to help induce relaxation and/or gain access to painful memories in treating individuals with Anxiety Disorders; moreover, according to some, it is particularly effective in treating individuals with Specific Phobia because they are highly hypnotizable. Finally, hypnosis is also commonly used in the treatment of medical conditions that have a psychological component such as asthma, chronic pain, obesity, etc.
Social Psychology
112. A peripheral message is more likely to alter a person's attitude if the receiver:
A feels happy
B feels bored
C feels angry
D devotes considerable thought to the content of the message
ANSWER "A"
According to the Elaboration Likelihood Model, persuasive messages can be communicated via a central route or a peripheral route. The central route is used when listeners think carefully about the contents of the message. When listeners do not think carefully about an argument's content, they rely more on superficial (peripheral) cues. Peripheral cues include the messengers' perceived attractiveness or expertise, and the receivers' mood. A positive mood, and in some cases fear, has been shown to increase the likelihood of attitude change. Advertisers and politicians often use strategies designed to elicit either a positive mood or fear in order to increase the persuasiveness of their message.
Neuropsychology
113. The effectiveness of thermal biofeedback as a treatment for migraine headaches is:
A less effective than relaxation therapy but more effective than electromyogram (EMG) biofeedback
B equally as effective as relaxation therapy
C more effective than relaxation therapy but less effective than electromyogram (EMG) biofeedback
D more effective than relaxation therapy
ANSWER "D"
Biofeedback, a form of complementary and alternative medicine (CAM) that falls under the category of mind-body therapies, refers to a technique that can give people better control over such body function indicators as blood pressure, heart rate, temperature, muscle tension, and brain waves. The National Institutes of Health (NIH) found thermal biofeedback, which measures skin temperature, to be more effective in treating migraines than relaxation therapy. EMG biofeedback, which measures muscle tension, is used as a treatment for tension headaches and data indicates it is equally effective as relaxation therapy. (See: National Institute of Neurological Disorders and Stroke (NIDS) of the National Institutes of Health (NIH). (2001). 21st Century Prevention and Management of Migraine Headaches. [Monograph]. Clinical Courier, 19(11).) .
Industrial/Organizational Psychology
114. Research has found that flexible work schedules have the greatest impact on:
A absenteeism
B job satisfaction
C productivity
D self-rated performance
ANSWER "A"
The results of a meta-analysis concluded that flextime has positive effects on employee absenteeism (i.e., improved attendance), productivity, satisfaction with work schedule, and job satisfaction. However, the greatest impact was on absenteeism (effect size = .93), followed by productivity (.45), satisfaction with schedule (.32), and job satisfaction (.15). Note that job satisfaction was lower than previously reported findings. Self-rated performance (D) was not significantly related to flextime (.04) [B. Baltes, T. Briggs, J. Huff, J. Wright, and G. Neuman, flexible and compressed workweek schedules: A meta-analysis of their effects on work-related criteria, Journal of Applied Psychology, 84(4), 1999, 496-513].
Clinical Psychology
115. Recent research on psychotherapy outcome indicates:
A 28% of therapy clients show marked improvement in symptoms after the 8th session
B 72% of therapy clients show marked improvement in symptoms after the 8th session
C 53% of therapy clients show marked improvement in symptoms after the 26th session
D 75% of therapy clients show marked improvement in symptoms after the 26th session
ANSWER "D"
Howard et al. (1986) reviewed the benefits that clients experienced due to attending therapy sessions as a function of the dosage of therapy sessions. Performing a meta-analysis of 15 previous outcome studies, their results verified that the longer patients remain in therapy, the greater their gains. They found that 15% of patients improve between intake and the first session (presumably due to spontaneous remission and the ameliorative effects of having sought treatment), 50% improve after eight sessions, and 75% improve after 26 sessions. Consistent with these findings, research by Kafta et al. (1994) found approximately 75% of the psychotherapy clients showed an alleviation of acute symptoms after the 28th session. (See: Howard, K. I., Kopta, S. M., Krause, M. S., and Orlinsky, D. E. (1986). The dose-effect relationship in psychotherapy. American Psychologist, 41, 159-164; Kopta, S. M., Howard, K. I., Lowry, J. L., and Beutler, L. E. (1994). Patterns of symptomatic recovery in psychotherapy. Journal of Consulting and Clinical Psychology, 62, 1009-1016.).
Developmental Psychology
116. Recently the relationship between Frank and one of his adult sons has become increasingly tense, conflicted and distant. Frank considers himself a loving and supportive father. He dismisses the problem as a temporary "phase" to avoid the situation. Frank’s attempt to explain away the problem, according to Whitbourne's self-concept model, is an attempt to maintain his own self-concept through:
A identity assimilation
B identity accommodation
C identity styles
D rationalization
ANSWER "A"
A. Identity process theory proposes that adjustment to aging can be conceptualized as involving the three processes of identity assimilation (maintaining self-consistency), identity accommodation (making changes in the self), and identity balance (maintaining a sense of self but changing when necessary). Despite the changes in the relationship with his son, Frank continues to think of himself as having a loving relationship in order to maintain his consistent sense of self. Research indicates only identity balance is positively related to internal state awareness, suggesting that the ability to incorporate age-related changes within an identity and at the same time maintain a consistent and positive view of the self is most conducive to successful aging. (See: Sneed, J.R. & Whitbourne, S.K. (2003). Identity Processing and Self-Consciousness in Middle and Later Adulthood. The Journals of Gerontology Series B: Psychological Sciences and Social Sciences, 58, 313-319.) .
Learning Theory and Behavior Therapy
117. Avoidance behavior is difficult to treat because:
A classical conditioning models don't apply.
B treatments require unacceptable levels of pain or other aversive applications.
C the feared stimulus is never presented.
D conditioning was done originally on an intermittent schedule.
ANSWER "C"
This is a familiar question. You might have seen something like it before. Once you change the terms into "English," it shouldn't be difficult. Basically it says that a fear (think of a phobia) is difficult to treat because you hardly ever confront it. If someone is afraid of bridges and never travels across a river on a bridge, you can never treat the person. Remember that treatments for phobias include at some point directly confronting the fear, through direct exposure to the feared object. If we avoid the object, we never get the chance to extinguish the reaction. None of the other alternatives comes as close to answering the question. As a test-taking strategy, it's a good idea to translate a question into "English" and give it a simple and exaggerated example. Then, find the answer that best explains your example.
Developmental Psychology
118. Normal aging is least likely to negatively affect:
A free recall
B cued recall
C working memory
D picture recognition
ANSWER "D"
You may have recalled that free recall is more affected by age than cued recall; however, picture recognition is the least demanding cognitive process of all of the choices, and is, therefore, the least affected by aging. [See D.C. Park, Mediators of long-term memory performance across the lifespan, Psychology and Aging, 1996, 11(4), 621-637].
Industrial/Organizational Psychology
119. An organization is considering different selection techniques and comparing the respective predictive validity of each. The validity of structured interviews as predictors of performance:
A are the most valid across different jobs
B are no more valid than unstructured interviews when used alone
C is increased when the interview includes both structured and unstructured items
D is increased when the interview is used in combination with a measure of general mental ability
ANSWER "D"
D. Summarizing the predictive validity of interviews and other selection techniques in a meta-analysis of the research, Schmidt and Hunter (1998) report measures of general mental ability are the most valid predictors across different jobs (a.). For structured and unstructured interviews, they note corrected validity coefficients of .51 and .38, respectively (b.). When an interview is combined with another selection procedure, especially a general mental ability test, the predictive validity of interviews is increased [The validity and utility of selection measures in personnel psychology: Practical and theoretical implications of 85 years of research findings, Psychological Bulletin, 124(2), 262-274]. .
Clinical Psychology
120. The effectiveness of Eye Movement Desensitization and Reprocessing (EMDR) in the treatment of Posttraumatic Stress Disorder is due to:
A exposure to anxiety-provoking stimuli
B changes in the functioning of the brain due to lateral eye movement stimulation
C unconditional positive regard
D placebo effect
ANSWER "A"
Eye Movement Desensitization and Reprocessing (EMDR) is a controversial therapeutic technique which combines techniques from cognitive-behavioral and other treatment approaches with lateral eye movements. Clients receiving EMDR are asked to recall anxiety-provoking memories and are then instructed to track the therapists' fingers which are moved from side to side. This process is repeated until the clients' anxiety is extinguished. Although Francine Shapiro, the developer of the technique, originally proposed that the bilateral eye movements accelerate the reprocessing of traumatic memories, a meta-analysis of 34 studies indicated that the eye movement component did not contribute to the effectiveness of EMDR. Rather, it appears that the element of exposure is responsible for its effects. The meta-analysis did find EMDR to be more effective than no-treatment or non-exposure treatments; however, it was no more effective than other exposure techniques. (P. R. Davidson, & K. C. Parker, Eye movement desensitization and reprocessing (EMDR): A meta-analysis. Journal of Consulting and Clinical Psychology, 2001, 69(2), 305-316).
Diagnosis and Psychopathology
121. The Personality Disorders vary in terms of their specific symptoms. However, DSM-IV identifies which of the following as a diagnostic criterion that they all share in common?
A an onset of symptoms during childhood
B a decline from a previous level of functioning
C significant distress or impaired functioning
D the individual is unaware of the maladaptive nature of his or her behavior
ANSWER "C"
When in doubt, "impaired functioning" is always a good guess for questions about the diagnostic criteria for a disorder since it is a requirement for many of the disorders included in DSM-IV. For the Personality Disorders, DSM-IV requires a history of symptoms by adolescence or early adulthood (not childhood), and it doesn't require a decline from a previous level of functioning or a lack of awareness about one's maladjustment.
Industrial/Organizational Psychology
122. Which of the following would be least useful as a method for management development?
A Outward Bound programs
B job rotation
C coaching or mentoring
D vestibule training
ANSWER "D"
Management development is a growing field in organizational psychology. A number of techniques are used to train (develop) managers including job rotation, coaching, mentoring, and even Outward Bound, which fosters trust and cooperation. Vestibule training involves the use of procedures or an environment similar to the actual job and is used for such jobs as bank tellers, clerks, and typists.
Test Construction
123. The ability of a test item to discriminate between members of different ability groups is best indicated by the __________ of its item characteristic curve?
A p-value
B slope
C x-intercept
D y-intercept
ANSWER "B"
Item characteristic curves (ICCs), which are associated with item response theory, are graphs that depict individual test items in terms of the percentage of individuals in different ability groups who answered the item correctly. For example, an ICC for an individual test item might show that 80% of people in the highest ability group, 40% of people in the middle ability group, and 5% of people in the lowest ability group answered the item correctly. An item's discrimination is indicated by the slope of its item characteristic curve. Items which have a steep slope do not discriminate very well, whereas, items with a flatter slope discriminate better. A graph and further description of ICCs can be found in the Test Construction chapter.
Test Construction
124. Cluster analysis would most likely be used to
A construct a "taxonomy" of criminal personality types.
B obtain descriptive information about a particular case.
C test the hypothesis that an independent variable has an effect on a dependent variable.
D test statistical hypotheses when the assumption of independence of observations is violated.
ANSWER "A"
The purpose of cluster analysis is to place objects into categories. More technically, the technique is designed to help one develop a taxonomy, or classification system of variables. The results of a cluster analysis indicate which variables cluster together into categories. The technique is sometimes used to divide a population of individuals into subtypes.
Clinical Psychology
125. Which of the following is NOT true regarding the Halstead-Reitan Neuropsychological Test Battery?
A It provides information on a range of cognitive strengths and weaknesses.
B It can usually be completed within 45 to 60 minutes.
C It must be administered by a highly trained examiner.
D The results are reflected in a combined score known as the Impairment Index.
ANSWER "B"
The Halstead-Reitan Neuropsychological Test Battery is a set of tests designed to assess attention, language, memory, abstract thinking, motor speed, and spatial reasoning (A). The purpose of the test is to provide an overall assessment of brain function. The battery must be administered by a highly trained examiner (C). The results of each subtest are combined into an Impairment Index (D). However, contrary to choice B, the battery usually requires 4 to 5 hours to administer.
Industrial/Organizational Psychology
126. In an attempt to increase motivation among employees, a company begins to pay employees based on their output. This will most likely result in:
A increased job satisfaction and increased quality of work
B decreased job dissatisfaction and increased quality of work
C decreased job dissatisfaction and increased quantity of work
D increased job satisfaction and no change in quantity of work
ANSWER "C"
If a person's pay is based on output, his or her output is likely to increase, but the quality of work will not necessarily increase. This makes sense -- if you are working so fast in order to produce more so that you can get paid more, the quality of your work is likely to suffer. In addition, increased pay is likely to decrease job dissatisfaction, as predicted by Herzberg's two-factor theory.
Ethics and Professional Practice
127. The difference between professional ethics and professional values is best stated by which of the following?
A If a psychologist is ethical, there should be no difference between ethics and values.
B Ethics are standards for practice set by the profession, while values refer to judgments of right and wrong.
C Ethics can be stated more specifically than values.
D Values specifically define appropriate professional conduct, while ethics are a more general code of proper professional conduct.
ANSWER "B"
Response B distinguishes between ethics and values. Values are concerned with what is good and desirable and ethics refer to correct or appropriate practice. Values and ethics are related in that the latter are usually derived from the former; for example, privacy is a value that is reflected in the ethical standard requiring psychologists to obtain clients' informed consent before releasing information about therapy. Responses A and C are not necessarily true, and answer D is not true since values do not specifically define appropriate professional conduct.
Test Construction
128. Cluster analysis would most likely be used to
A construct a "taxonomy" of criminal personality types
B obtain descriptive information about a particular case
C test the hypothesis that an independent variable has an effect on a dependent variable
D test statistical hypotheses when the assumption of independence of observations is violated
ANSWER "A"
The purpose of cluster analysis is to place objects into categories. More technically, the technique is designed to help one develop a taxonomy, or classification system of variables. The results of a cluster analysis indicate which variables cluster together into categories. The technique is sometimes used to divide a population of individuals into subtypes.
Learning Theory and Behavior Therapy
129. Lewinsohn’s behavioral model hypothesized depression is the result of:
A a distorted perception of reality
B low self-esteem and pessimism
C a high rate of response-contingent punishment
D a low rate of response-contingent reinforcement
ANSWER "D"
D. Based on operant conditioning, Lewinsohn’s theory proposes depression is associated with a low rate of response-contingent positive reinforcement. A low rate of reinforcement results in a low rate of social and other behaviors due to a lack of reinforcement which essentially extinguishes the contingent behaviors. The model also proposes the low rate of reinforcement elicits depressive behaviors, such as dysphoria, fatigue, and other somatic symptoms as well as cognitive symptoms such as low self -esteem, pessimism, and feelings of guilt. (See: Lewinsohn, P. M. (1974). A behavioral approach to depression. In R. J. Friedman & M. M. Katz (Eds.), Psychology of depression: Contemporary theory and research (pp. 157-178). Oxford, England: John Wiley & Sons.) .
Neuropsychology
130. An infant has a slowed heart rate while sleeping. The infant has
A apnea.
B SIDS.
C bradycardia.
D tachycardia.
ANSWER "C"
Bradycardia means a slow heart rate. Apnea means difficulty breathing while sleeping. SIDS stands for Sudden Infant Death Syndrome. Tachycardia refers to rapid heart rate.
Social Psychology
131. A person's reactions to stress are most negative when the stress is unexpected. Moreover, a person's reactions are most affected by
A his or her actual capacity to cope with the stress.
B his or her beliefs regarding his or her ability to cope with the stress.
C the total number of stressors.
D the actual availability of adequate social support.
ANSWER "B"
It is a person's perception of control or ability to cope that determines his or her response to stress. When a person believes he or she has no control over the situation or does not have the necessary coping mechanisms, stress will have a more detrimental effect.
Ethics and Professional Practice
132. A psychologist discovers a client is HIV positive, has not informed his/her partner and is having unsafe sex with the partner. Current laws related to the psychologist’s obligation is most accurately described by which of the following?
A Health professionals are legally required to breach confidentiality in this situation.
B Health professionals are legally required to NOT breach confidentiality in this situation.
C Health professionals are legally required to breach confidentiality in this situation in some states and prohibited in others.
D Health professionals are legally allowed to determine whether or not to breach confidentiality in this situation.
ANSWER "C"
C. In this situation, the laws regarding breach of confidentiality vary from state to state and are inconsistent. Some states have laws that prohibit practitioners from notifying the partners of HIV/AIDS clients, and some state laws require practitioners to inform the partners.
Developmental Psychology
133. For the prevention and treatment of childhood sleep disorders, Ferber recommends that by six months parents should:
A provide only verbal comfort or a pat on the back if the child cries
B rock the child until he/she falls asleep
C feed the child before putting him/her to bed or provide the child with a bottle at bedtime
D be flexible regarding the child's natural sleep cycle
ANSWER "A"
Dr. Richard Ferber, along with other experts, recommend a progressive waiting period to help children fall asleep and stay asleep. The method, sometimes referred to as "Ferberizing" involves putting the baby to bed while he or she is still awake. If the child cries, parents are instructed to wait a predetermined interval –usually five minutes. At that time they are instructed to go in to check and comfort the child verbally, but without rocking, feeding, or picking up the child. After doing this, they are to leave the room. If the child cries again, they are to wait 10 minutes before providing the same response. After each successive time the interval is increased until the child falls asleep. Contrary to Choice "D," Ferber recommends sticking to a regular schedule; however, he does acknowledge that under certain circumstances, for instance, if the child is sick or you are traveling, you may need to be flexible regarding the schedule.
Diagnosis and Psychopathology
134. The cause of anxiety is often
A misunderstood fear.
B flat affect.
C anger turned inward.
D depression.
ANSWER "A"
Anxiety is sometimes defined as a fear that is not understood by the individual.
Diagnosis and Psychopathology
135. The best systematic method for tracking the progression of Alzheimer's Disease is
A daily observation of changes in the patient's mood and behavior.
B reports from significant others and family members.
C obtaining self-reports from the patient.
D administering assessments of cognitive functioning on a regular basis.
ANSWER "D"
This is one of those questions that may drive you crazy, because all four choices describe ways in which an Alzheimer's patient can be assessed. Thus, you must choose the best answer. Because Dementia of the Alzheimer's Type is a cognitive disorder, it makes sense that the most direct way to assess it on an ongoing basis is to regularly administer cognitive assessments. The question is really as much about how Alzheimer's is defined as it is about how to assess it.
Neuropsychology
136. Recent studies investigating ethnicity and psychotropic medication dosage have found:
A Therapeutic and side effect differences are not related to race/ethnicity
B Caucasians and Asians experience the same severity of side effects from the same dose, although Caucasians require lower doses to obtain the same therapeutic effects.
C Caucasians experience more severe side effects from the same dose than Asians and require lower doses to obtain the same therapeutic effects
D Asians experience more severe side effects from the same dose and require lower doses to obtain the same therapeutic effects
ANSWER "D"
D. Research indicates individuals respond differently to psychotropic medications based on their ethnic background or biological and environmental differences in ethnicity. In particular, evidence suggests Asian patients, as a group, metabolize medications like psychotropics more slowly than Caucasian patients and therefore are more sensitive to the therapeutic and side effects. Based on these findings, researchers recommend the starting dosage level for Asian patients be lower than the standard dosage for Caucasian patients. (See: Lin, K. M. & Cheung, F. (1999) Mental health issues for Asian Americans, Psychiatric Services, 50(6), 774-780. and Lin, K.M.. & Smith, M.W. (2000). Psychopharmacotherapy in the Context of Culture and Ethnicity in Ethnicity and Psychopharmacology. Edited by Pedro Ruiz ( Review of Psychiatry Series, 19(4). Oldham, J.O. and Riba, M.B., (Eds.), Washington, DC, American Psychiatric Press, 1-27) .
Community Psychology
137. Which of the following is the best conclusion regarding repressed memories of sexual abuse?
A Based on what we know about the process of memory, the idea that it's possible to forget but later recall childhood sexual abuse is not credible.
B Some people may forget about sexual abuse and subsequently remember it later in life. However, as of yet, no factors that distinguish between these people and other victims of sexual abuse are known.
C Some people, particularly less intelligent victims, may forget about sexual abuse and subsequently remember it later in life.
D Due to the traumatic nature of sexual abuse, memories of it are repressed in the majority of cases.
ANSWER "B"
In a study conducted by Elizabeth Loftus and her colleagues (who has been critical of the possibility of recovering repressed sexual abuse memories), it was found that 19% of sexual abuse victims had forgotten but later recalled sexual abuse. Although this is a lower rate than reported in other studies (which the authors criticize as methodologically flawed), the authors did conclude that, at least for some women, repression and recovery of these memories are possible. No factors distinguishing these women from victims who don't repress memories of sexual abuse were found.
Industrial/Organizational Psychology
138. Holland's approach to vocational guidance is based on the assumption that:
A interests motivate occupational choice
B behavior is a function of personality-environment congruence
C job satisfaction and performance are related to a match between aptitude and job requirements
D interests change in a predictable way over the course of the lifespan
ANSWER "B"
Holland's six occupational themes represent both personality and environment types. According to Holland, job satisfaction, productivity, and other behaviors are affected by the congruence or fit between a person's personality and the nature of the work environment.
Industrial/Organizational Psychology
139. Individuals vulnerable to burnout tend to be:
A lazy workers with low motivation.
B insecure hard workers.
C fulfilled from sources outside the job.
D managers with responsibility and power.
ANSWER "B"
Burnout is most suffered by compulsive workaholics who are insecure and unfulfilled in their personal lives. Specifically, burnout victims are not lazy, nor are they fulfilled in their outside lives. Finally, they tend to be in dead-end jobs with little hope for promotion. Knowing these facts (which are reviewed in the Industrial/Organizational section), you could have deduced the correct answer.
Learning Theory and Behavior Therapy
140. Poor recall for information learned yesterday as the result of learning that took place last month is referred to as:
A retroactive interference
B proactive interference
C retrograde amnesia
D anterograde amnesia
ANSWER "B"
The prefixes ("pro" and "retro") indicate which material is being interfered with. Proactive interference or inhibition occurs when previous learning interferes with new learning. Retroactive interference or inhibition occurs when new learning interferes with previous learning. Retrograde amnesia (C) is a failure to remember events that occurred prior to a trauma. And anterograde amnesia (D) is an impaired ability to learn new information.
Learning Theory and Behavior Therapy
141. If you are going to be tested on the definitions of unrelated words at 8:00 in the morning two weeks from today, you should keep in mind when deciding when to study that:
A retroactive interference is more of a concern if you study the night before the exam rather than the week before.
B retroactive interference is more of a concern if you study the week before the exam rather than the night before.
C proactive interference is more of a concern if you study the week before the exam rather than the night before.
D retroactive and proactive interference are not a problem with this kind of learning.
ANSWER "B"
To answer this question, you need to understand what retroactive and proactive interference are. Retroactive interference occurs when your ability to recall X is difficult because of interference by something you learned after X. The longer the period of time between learning X and being tested on it, the greater the opportunity for retroactive interference. Proactive interference occurs when the ability to recall X is impaired by previously learned material. Proactive interference can occur regardless of how long the interval is between learning X and recalling it. Finally, retroactive and proactive interference are most likely to be a problem for information that is not inherently meaningful, which would be the case for a set of unrelated words.
Neuropsychology
142. Medications used to treat symptoms of Parkinson's Disease block the effects of:
A dopamine
B serotonin
C acetylcholine
D GABA
ANSWER "C"
If you incorrectly chose "A," remember that Parkinson's is characterized by insufficient amounts of dopamine; thus, medical treatment seeks to increase dopamine, not block it. Another treatment strategy is to block acetylcholine receptors to help restore the normal balance between dopamine and acetylcholine. In Parkinson's, the loss of dopamine results in a relative excess of acetylcholine. Medications aim to restore the balance by increasing dopamine and/or blocking acetylcholine receptors.
Neuropsychology
143. Right-left confusion is a characteristic of damage to the:
A temporal lobe
B frontal lobe
C occipital lobe
D parietal lobe
ANSWER "D"
Left-right disorientation is typically caused by damage in the parietal lobe, specifically the left angular gyrus located between the parietal and temporal lobes. It is also one of the symptoms of Gerstmann’s syndrome.
Community Psychology
144. The best example of a secondary prevention program is
A a rehabilitation program.
B crisis intervention.
C a community education program.
D Head Start.
ANSWER "B"
Secondary prevention involves early detection and intervention for a problem in order to reduce its duration and keep it from getting worse. Crisis intervention, suicide hotlines, and screening tests are all examples of secondary prevention.
Diagnosis and Psychopathology
145. Briquet’s syndrome is also known as a:
A Conversion Disorder
B Somatization Disorder
C Body Dysmorphic Disorder
D Hypochondriasis
ANSWER "B"
B. Termed after the physican who described the condition in the 1850s, Briquet’s syndrome, or Somatization Disorder, is a chronic Somatoform Disorder with multiple physical symptoms that cannot be explained entirely by a general medical condition or the effects of a substance. The other three response choices are also Somatoform Disorders.
Clinical Psychology
146. From the perspective of psychoanalytic theory, mania represents:
A a biological illness that psychoanalysis can do nothing about.
B acting-out of libidinous impulses.
C a regression to an infantile state.
D a defense against depression.
ANSWER "D"
According to classical psychoanalytic theory, mania occurs as a defense against depression, due to an inability of the person to tolerate or admit to being depressed.
Clinical Psychology
147. According to recent meta-analyses of child psychotherapy outcome studies, which of the following statements is most true?
A There are no differences between the effectiveness of behavioral and non-behavioral interventions in the treatment of children.
B Girls respond better to psychotherapy than boys, with adolescent girls responding best.
C Girls respond better to psychotherapy than boys, with younger girls responding best.
D At all age levels, boys respond better to psychotherapy than girls.
ANSWER "B"
This is one of the many areas where the results of research are contradictory and somewhat controversial. However, the most recent meta-analyses of research in this area have found that across treatment approaches, girls respond better than boys, with adolescent girls responding best of all. This is somewhat contradictory to earlier research, which found that younger children respond better than older children.
Because the findings of research sometimes contradict each other, it can be frustrating to decide which is the best answer to questions such as these. Of course, you'd want to find an answer that is consistent (or not inconsistent, at least) with the results of all the research. However, if such a response is not available, you should go with the results of more recent research.
Developmental Psychology
148. According to Huesmann et al. (2003), the long-term effects of TV violence on children growing up in the 1970-80’s, indicate childhood exposure to media violence:
A predicts young adult aggressive behavior only for males
B predicts young adult aggressive behavior for males and females
C predicts young adult aggressive behavior only for males with low social economic status or intellectual ability
D predicts young adult aggressive behavior only for males and females with low social economic status or intellectual ability
ANSWER "B"
Unlike studies on children growing up in the 1960’s which found aggressive behavior only in males, Huesmann et al. (2003) found childhood exposure to TV violence is stimulating an increase in adult aggression in males and females. This effect persists even when the effects of socioeconomic status, intellectual ability, and a variety of parenting factors are controlled. More childhood exposure to TV violence, greater childhood identification with same-sex aggressive TV characters, and a stronger childhood belief that violent shows tell about life “just like it is” predicted more adult aggression regardless of the initial aggressiveness of the child. (See: Rowell Huesmann, L., Moise-Titus, J., Podolski, C., and Eron, L.D. (2003). Longitudinal Relations Between Children’s Exposure to TV Violence and Their Aggressive and Violent Behavior in Young Adulthood: 1977–1992. Developmental Psychology, 39(2), 201–221.).
Developmental Psychology
149. A phoneme is
A the smallest unit of language
B the smallest unit of language which carries meaning
C a whole word, which, when combined with gestures and intonation can express an entire thought.
D another way of saying "call me"
ANSWER "A"
A phoneme is the smallest unit of language. Phonemes can be single letters or two letters combined which distinguish one word from another. For example, the letters "m" and "b" distinguish "met" from "bet." Thus, a phoneme can distinguish the meaning of different words, but by itself, does not carry meaning. A morpheme (Choice B), on the other hand, is the smallest unit of language which carries meaning. For example, the word "reading" consists of two morphemes: "read" and "ing" (the "ing" indicates an active process). Choice "C" is a description of a holophrase. Choice "D" is just plain silly.
Community Psychology
150. Which of the following is most consistent with the research regarding childhood sexual abuse?
A the effects are less severe for younger children compared to older children
B the effects are less severe the older the perpetrator is at the time of the abuse
C the effects are less severe when the perpetrator is a family member
D the effects are less severe when the perpetrator is a stranger
ANSWER "D"
Childhood sexual abuse often results in long-term consequences for the victims; however, certain factors have been found to moderate the severity of consequences. For example, the effects are generally less severe when the perpetrator is a stranger rather than a family member. However, approximately 70% of all incidents of childhood sexual abuse are committed by a family member.
Clinical Psychology
151. Johnny, age 10, teaches his brother Justin, age 5, how to ride a skateboard. This interaction can best be described as:
A Parallel process
B Complementary communication
C Elaborative rehearsal
D Symmetrical communication
ANSWER "B"
Communication patterns can be characterized as either symmetrical or complementary. In symmetrical communication there is equality between the partners. In complementary communication there is inequality, with one member taking a dominant role and the other a subordinate role. In this example, Johnny, the older brother, took on the dominant teacher role and his younger brother was in the subordinate role of student. Although completely unrelated to this model, you should also be familiar with the other two terms -- parallel process (A) and elaborative rehearsal (C). Parallel process is a phenomenon in clinical supervision where the therapist in training behaves toward the supervising therapist in ways that mirror how the client is behaving toward the therapist in training. Elaborative rehearsal involves thinking about how new information relates to existing memory. .
Diagnosis and Psychopathology
152. From Wolpe's classical conditioning perspective, neurotic depression:
A is a conditioned response that can be alleviated through extinction trials in which the neutral (conditioned) stimulus is repeatedly presented without the depression-inducing (unconditioned) stimulus.
B is a response to anxiety and can, therefore, be alleviated by using systematic desensitization to eliminate the anxiety.
C is due to attributional biases that, through conditioning, have become associated with certain types of events and can be eliminated through reattribution training.
D results when there is an absence of response contingent reinforcement and is best treated by counterconditioning in which depression is paired with a variety of pleasure-producing (unconditioned) stimuli.
ANSWER "B"
Even if you are unfamiliar with Wolpe's explanation of depression, you may have been able to pick the right answer to this question as long as you have him associated with systematic desensitization. Wolpe distinguished between several types of depression. He linked neurotic depression to anxiety and considered systematic desensitization to be an effective treatment.
Research Design and Statistics
153. A significant finding for a one-way ANOVA indicates that the
A group means were different.
B sample means were different.
C population means were different.
D within-group variance was different.
ANSWER "C"
We use statistical tests to make inferences about a population. So if we have significant results, we assume that this represents what happens in the real world -- that is, in the population.
Diagnosis and Psychopathology
154. Which of the following smoking cessation treatments is least effective for long-term abstinence?
A hypnosis
B acupuncture
C aversive techniques
D nicotine replacement
ANSWER "D"
Nicotine replacement interventions (e.g., nicotine gum, nicotine patch) have not been found to be very effective over the long-term unless they are combined with other treatment interventions. In a meta-analysis based on over 600 smoking cessation studies, the mean quit rate (based on short-term and long-term rates) for hypnosis = .36; acupuncture = .30; aversive techniques = .27; nicotine gum = .16; and control group = .06. [C. Viswesvaran and F. L. Schmidt, A meta-analytic comparison of the effectiveness of smoking cessation methods, Journal of Applied Psychology, 1992, 77(4), 554-561]. Another meta-analysis that addressed the combination of nicotine gum with other treatment interventions but compared short-term to long-term outcomes also concluded that nicotine gum by itself is not very effective in the long-term, but is effective when combined with other treatment interventions [A. Cepeda-Benito, Meta-analytical review of the efficacy of nicotine chewing gum in smoking treatment programs, Journal of Consulting and Clinical Psychology, 1993, 61(5), 822-830].
Industrial/Organizational Psychology
155. According to social learning theory, job training would be most effective when using:
A behavioral modeling
B vestibule training
C reinforcement on a variable interval schedule
D reinforcers tailored to each employee's needs
ANSWER "A"
According to Bandura's social learning theory (or theory of observational learning), it is possible to learn a given behavior merely by watching a model perform it. Behavioral practice and reinforcement, though they influence the probability that a behavior will be learned, are not absolutely necessary for the behavior to be imitated. Therefore, Choices C and D, which involve reinforcement procedures, can be eliminated. Vestibule training (B) involves a simulation of work conditions, although it does not necessarily involve modeling.
Ethics and Professional Practice
156. Baumrind in 1964 indicated that whose study was unethical?
A Bandura's
B Zimbardo's
C Milgram's
D Harlow's
ANSWER "C"
Baumrind wrote the article, "Some thoughts on the ethics of research: After reading Milgram's behavioral study of obedience." American Psychologist 19 421-423. Milgram's study would be considered unethical today as would Zimbardo's. It is also doubtful that parents would be keen on having their children watch and imitate aggressive models, and the animal rights people would object to Harry Harlow's techniques.
Neuropsychology
157. For patients who develop a disturbing degree of hand tremor as the result of taking lithium carbonate, which of the following would be useful?
A increasing the dosage of lithium
B administering propranolol (Inderal)
C administering an SSRI
D administering L-tryptophan
ANSWER "B"
Lowering the dose of lithium ordinarily eliminates tremors, but you're not given that option. Propranolol is a beta-blocker, which reduces the physiological aspects of anxiety, and, so it makes sense, that it would be useful for reducing tremors.
Ethics and Professional Practice
158. Prior to conducting a court-ordered evaluation of a minor, you:
A must obtain informed consent from the minor.
B must obtain informed consent from the parents or legal guardians of the minor.
C must insure that the parents or legal guardians are present during the evaluation.
D should discuss the purpose of the evaluation with the minor.
ANSWER "D"
APA's "Specialty Guidelines for Forensic Psychologists" states that "psychologists obtain informed consent unless the service is court ordered." Thus, regardless of whether the examinee is an adult or a minor, in a court-ordered evaluation, it is not necessary to obtain informed consent. However, the subject of an evaluation should always be informed of the purpose, methods, and intended use of the evaluation.
Learning Theory and Behavior Therapy
159. The basic requirements of a token economy are:
A stimulus sensitization, choice of tokens, rate of exchange.
B target behaviors, choice of reinforcers, rate of exchange.
C goal setting, staff cooperation, choice of reinforcers.
D target behaviors, choice of tokens, primary reinforcers.
ANSWER "B"
To institute a token economy program, you need to know the behaviors you want to change (the target behaviors). You also need to know what is reinforcing for the client (choice of reinforcer). For a hospitalized schizophrenic, it might be walking around; for a child, it might be a candy treat. You also need to know the relationship between token and the reinforcer (rate of exchange); that is, how many tokens will purchase the reinforcer.
Developmental Psychology
160. Research examining methods of informing children with cancer about their illness suggests that
A providing young children with limited information is best since they are unable to understand the illness and only become confused when given details about it.
B limiting information to responses to children's direct questions is better than "volunteering" details about their illness or treatment.
C children are better off when they are given information about their diagnosis, prognosis, and treatment procedures.
D it is best to let the parents decide what and when to tell children about their illness.
ANSWER "C"
The research on this issue has generally found that open discussions with children about their illness are associated with the best psychosocial outcomes. Of course, parents are ultimately responsible for deciding what and when to tell children (response D), but a professional familiar with the research can at least inform them of the findings to help them decide what to do. See D. J. Bearison, Pediatric psychology and children's medical problems, in W. Damon (ed.), Handbook of Child Psychology (Vol. 4), New York, John Wiley & Sons, 1998.
Social Psychology
161. Studies investigating the social cognition correlates of aggression in children suggest that the cognitive biases of these children tend to create a
A self-fulfilling prophecy effect.
B contrast effect.
C personal fable.
D rebound effect.
ANSWER "A"
Research by N. R. Crick and K. A. Dodge (A review and reformulation of social information-processing mechanisms in children's social adjustment, Psychological Bulletin, 1994, 115,74-101), for example, found that aggressive children often misinterpret the ambiguous or prosocial behaviors of peers as hostile in intent and respond in aggressive ways. The peers then respond negatively, thereby, creating a self-fulfilling prophecy effect.
Research Design and Statistics
162. You have conducted a study assessing the relationship between salary and job performance, and you find a significant correlation between these two variables. Your assistant tells you that the data fail to take into account a $25.00 cost of living raise which every employee received. You should:
A decide that the raise invalidated the research.
B reanalyze the data after the raises have been added to the current salary.
C not worry about small details; the actual amount is too small to make a significant difference.
D assume the correlation will not be affected.
ANSWER "D"
The basic point being tested here is that if you add a constant to each score -- in either or both data sets -- the relationship between the two variables won't be affected. In other words, adding a constant to every score does not affect the correlation coefficient. The same is true of multiplying or dividing all scores by a constant, or subtracting a constant from every score.
Neuropsychology
163. The concept of long-term potentiation is most closely related to
A sleep.
B memory and learning.
C hunger and thirst.
D sensory perception.
ANSWER "B"
The term long-term potentiation refers to a facilitation or increase in synaptic efficiency that is long lasting (i.e., hours to days). Long-term potentiation was first discovered in the hippocampus and seems to have the most concrete applications to the processes of learning and memory. It has been theorized that learning, through stimulation of certain hippocampal neurons, results in long-term potentiation and thereby facilitates those neurons' receptivity to further stimulation.
Clinical Psychology
164. Biofeedback is often part of treatments aimed at lowering tension and arousal and increasing relaxation. For this purpose, biofeedback is being used to control the:
A sympathetic nervous system.
B parasympathetic nervous system.
C somatic nervous system.
D endocrine system.
ANSWER "B"
Biofeedback and other stress/anxiety reducing techniques are designed to induce a positive parasympathetic state since it is the parasympathetic nervous system that is responsible for rest, relaxation, and recuperation. On the other hand, if you wanted to increase blood flow to change body temperature, biofeedback would affect the sympathetic nervous system.
Neuropsychology
165. Glutamate is a neurotransmitter associated with
A alcohol cravings.
B dietary changes.
C cognitive abilities which are correlated with alcohol use.
D social ability which is correlated with alcohol use.
ANSWER "C"
Glutamate is a neurotransmitter that is associated with cognitive functions, more specifically learning and memory.
Ethics and Professional Practice
166. A psychologist determines clients’ fees based on their current income. This “sliding scale” practice is:
A explicitly prohibited in the Ethics Code
B explicitly recommended in the Ethics Code
C unacceptable but not explicitly mentioned in the Ethics Code.
D acceptable but not explicitly mentioned in the Ethics Code.
ANSWER "D"
D. As long as they are fair and serve the best interests of the client, sliding scale fees are generally considered acceptable. The Ethics Code does not explicitly address sliding scale fees.
Neuropsychology
167. In most patients, the earliest signs of Huntington's disease are
A apathy and disorientation to time and place.
B depression and forgetfulness.
C athetosis and chorea.
D slowed movements and resting tremor.
ANSWER "B"
In Huntington's disease, personality and cognitive changes often precede motor symptoms. Athetosis and chorea (answer C) are characteristic of this disorder but are late-appearing. Slowed movements and resting tremor (answer D) are characteristic of Parkinson's disease.
Industrial/Organizational Psychology
168. A researcher studying the relationship between age and job satisfaction collects data for a group of young, middle-aged, and older workers. The researcher can expect to find that
A the young people are most satisfied with their jobs.
B the middle-aged people are most satisfied with their jobs.
C the older people are most satisfied with their jobs.
D there is no difference between the three groups in terms of their job satisfaction.
ANSWER "C"
One of the most stable findings in job satisfaction research is that age is positively correlated with satisfaction. The older the employee, the higher the level of satisfaction he or she tends to report.
Neuropsychology
169. The Sickness Impact Profile is used to:
A assess the impact of disease on physical and emotional functioning
B assess emotional reactions to chronic illness
C diagnose physical illnesses
D diagnose personality disorders
ANSWER "A"
The Sickness Impact Profile (SIP) is one of the most comprehensive quality of life measures available. It is used to assess the impact of disease on both physical and emotional functioning. Its focus is on behavioral measures of daily living such as sleep, eating, social interactions, and emotional behaviors. Contrary to "B," however, it is not used to assess emotional or other subjective reactions to illness.
Clinical Psychology
170. The term "best practices" refers to:
A empirical evidence to support effectiveness of counseling approaches
B counselor characteristics that contribute to increased effectiveness.
C matching certain client problems with specific counseling models
D clinical decision making accountability and concerns for service costs
ANSWER "A"
A. The term "best practices" is defined as approaches to counseling practice that have empirical evidence to support their effectiveness. It is one way managed care companies and practitioners strive to maintain accountability for costs of services and clinical decision making, as well as ascertain the most effective practices available to individuals.
Learning Theory and Behavior Therapy
171. Of the following, which is the best way to facilitate memory of something you have recently learned?
A sleep
B study related material
C study unrelated material
D engage in strenuous physical activity
ANSWER "A"
Research has demonstrated that recall of a particular piece of information is greater after a given time period of sleep than after the same period of wakeful activity. This finding lends support to the interference theory of forgetting, which proposes that forgetting of a given piece of information is caused by interference from other information. When one is sleeping, there is less potential for any such interference to occur.
Ethics and Professional Practice
172. Dr. Bean is a newly licensed clinical psychologist who is practicing in a small rural town. She is the only mental health practitioner in a 150-mile radius. Dr. Bean finds that some of her clients have mental health problems that she has little or no experience treating. Her best course of action is to
A refuse to treat the clients who have problems that she is unfamiliar with.
B obtain appropriate training before treating the clients who have problems that she is unfamiliar with.
C inform the clients of her lack of experience and let them decide what to do.
D see the clients but obtain supervision or consultation by telephone.
ANSWER "D"
According to Ethics Standard 2.01(d), “When psychologists are asked to provide services to individuals for whom appropriate mental health services are not available and for which psychologists have not obtained the competence necessary, psychologists with closely related prior training or experience may provide such services in order to ensure that services are not denied if they make a reasonable effort to obtain the competence required by using relevant research, training, consultation, or study.” Since Dr. Bean is the only mental health professional in town, this is the best course of action of those given in the responses since there is no indication that the clients have problems that Dr. Bean doesn't want to treat or that she would not be able to treat with consultation or supervision.
Learning Theory and Behavior Therapy
173. Once a behavior has been established through operant conditioning, the behavior would most likely ________________ immediately following the removal of the reinforcer.
A increase
B decrease gradually
C decrease rapidly
D remain the same
ANSWER "A"
In operant extinction, removal of a reinforcer does not result in an immediate decrease of the behavior. In fact, what occurs initially is a temporary increase in behavior, which is known as a "response burst." .
Diagnosis and Psychopathology
174. Approximately 50% of children with Autism fail to speak or develop language skills. Those that do speak are most likely to show
A pronoun reversal.
B phoneme omissions.
C phonological errors.
D syllable reversal.
ANSWER "A"
Children with Autistic Disorder are often mute and, if they speak, they exhibit a number of language abnormalities. A common abnormality is echolalia; another is the reversal of pronouns (e.g., using "you" instead of "I").
Ethics and Professional Practice
175. A defendant's ability to take part in and understand the proceedings against him in a criminal trial is an issue of:
A sanity
B competence
C consent
D testamentary capacity
ANSWER "B"
The situation described in the question refers to competence to stand trial.
Developmental Psychology
176. Which of the following of Kohlberg's stages of moral development is in the correct order?
A Punishment and obedience orientation, social contract and individual rights orientation, instrumental hedonistic orientation, authority and social order maintaining orientation
B Preconventional, postconventional, conventional
C Social relations orientation, instrumental hedonistic orientation, individual rights orientation, social order maintaining orientation
D Instrumental hedonistic orientation, social relations orientation, social contract and individual rights orientation, universal ethical principles orientation
ANSWER "D"
This question is an example of EPPP questions that are mostly about reading comprehension. If you were able to stay awake through all the choices, you would have recognized Stage 2, 3, 5 and 6 as the correct selection.
School Psychology/Intelligence
177. A Performance IQ score that is 15 points higher than the Verbal IQ score on the WAIS-III is suggestive of all of the following except
A learning disabilities.
B low socioeconomic status.
C depression.
D antisocial behavior.
ANSWER "C"
A Verbal-Performance IQ discrepancy on the Wechsler test is considered significant when it is 12 points or more. There are a number of reasons why the Performance IQ may be higher than the Verbal IQ. The factors listed in answers A, B, and D may account for a higher Performance IQ, while depression (answer C) is a possibility when the Verbal IQ is higher.
Industrial/Organizational Psychology
178. From an organizational systems approach, positive feedback is most useful as an intervention in order to:
A provide information which helps plan for corrective actions.
B reinforce adaptive responses.
C extinguish maladaptive organizational interactions.
D establish productive work groups.
ANSWER "A"
In any system, feedback is useful to make corrections. Even our nervous system makes use of this phenomenon: as your body temperature rises on a cold morning, you stop shivering. Likewise with institutions and organizations. This question asks about positive feedback, which, in systems terminology, refers to information that encourages disruption from the status quo. In organizational consulting and family therapy, positive feedback is used to correct or change a pattern; in response to positive feedback the system deviates from homeostasis, or the status quo. This question is not about reinforcing other patterns or extinguishing maladaptive patterns, as two of the choices indicate. These terms are related to behavioral learning theory, not systems theory.
Social Psychology
179. A man says that if he goes to watch his favorite baseball team play, his team will lose, despite the team's excellent record. This man's statement is an example of a(n):
A delusion
B fundamental attribution bias
C illusory correlation
D self-fulfilling prophesy
ANSWER "C"
An illusory correlation is the tendency to overestimate the association between variables that are uncorrelated or only slightly correlated. Because the man's presence at the baseball game is unlikely to affect the outcome, his belief that it would cause the team to lose would be considered an illusory correlation. There is insufficient evidence to indicate that the man's statement represents a delusion (A), which is "a false belief based on incorrect inference about external reality that is firmly sustained despite what almost everyone else believes....and obvious proof or evidence to the contrary" (DSM-IV-TR, p. 821). A fundamental attribution bias (B) refers to the tendency to overestimate dispositional (personality) factors and underestimate situational factors in explaining the behavior of others. Self-fulfilling prophesy effect (D) occurs when our expectations about another person affect that person's behavior.
Test Construction
180. The purpose of rotation in factor analysis is to facilitate interpretation of the factors. Rotation:
A alters the factor loadings for each variable but not the eigenvalue for each factor
B alters the eigenvalue for each factor but not the factor loadings for the variables
C alters the factor loadings for each variable and the eigenvalue for each factor
D does not alter the eigenvalue for each factor nor the factor loadings for the variables
ANSWER "C"
C. In factor analysis, rotating the factors changes the factor loadings for the variables and eigenvalue for each factor although the total of the eigenvalues remains the same.
Ethics and Professional Practice
181. During an initial interview with a psychotherapy client, the client tells the psychologist that he wants to pay for his sessions himself without the psychologist releasing any information to his insurance company. He also requests that the psychologist not keep any records of their sessions. Agreeing to this arrangement would be:
A ethical and legal, as long as the psychologist keeps a record of the financial payments and reports those amounts on income tax forms
B ethical, as long as the psychologist determines that the client's reasons for the request are valid
C unethical, because psychologists cannot withhold records from a client's insurance company
D unethical and possibly illegal, because psychologists must maintain records
ANSWER "D"
According to APA's Record Keeping Guidelines (1993, 48(9), 984-986) records should be maintained and should minimally include: identifying data, dates and types of service, fees, any release of information obtained, any assessment, plan for intervention, consultation, summary reports, and/or testing reports. Although these guidelines are aspirational, they have become a standard of care for psychologists and, therefore, not maintaining any records about a client would be considered unethical. Many states also require the maintenance of client records, and therefore, not keeping records may be illegal. In regards to Choice C, clients generally have the right not to permit the release of records to their insurance company if the insurance company will not be responsible for payment. However, if the psychologist is a contracted provider for the insurance company, the contract may include restrictions on providing "out-of-pocket" treatment to its members or it may require the client to sign a waiver releasing the insurance company of its financial responsibility.
Diagnosis and Psychopathology
182. The preferred treatment for Agoraphobia is
A implosive therapy.
B systematic desensitization
C in-vivo exposure with response prevention.
D participant modeling.
ANSWER "C"
Much research supports the generally accepted conclusion that in-vivo exposure with response prevention is the preferred psychological treatment for Agoraphobia.
Neuropsychology
183. A physician refers a patient to you with temporal lobe damage. You would be least likely to expect your patient to exhibit
A problems understanding what you say.
B changes in sexual behavior.
C anterograde amnesia.
D impulsive statements.
ANSWER "D"
A patient making impulsive statements or other types of problems with response inhibition is more from the result of damage to the frontal lobes. Answer "A" (problems with auditory perception), "B" (changes in sexuality), and "C" (severe anterograde amnesia) are examples of behaviors that have been linked to temporal lobe damage.
Neuropsychology
184. A substance that stimulates or mimics the effects of a neurotransmitter is referred to as:
A agonistic
B antagonistic
C cholinergic
D dopaminergic
ANSWER "A"
An agonist or agonistic drug is one that facilitates the effects of a neurotransmitter. An antagonist (B) blocks or inhibits the effects of a neurotransmitter. Cholinergic (C) refers to synaptic transmission that is mediated by the release of acetylcholine. Dopamine (D) refers to synaptic transmission mediated by the release of dopamine.
Test Construction
185. A measure of relative strength of a score within an individual is referred to as a(n):
A ipsative score
B normative score
C standard score
D independent variable
ANSWER "A"
Ipsative scores report an examinee's scores using the examinee him or herself as a frame of reference. They indicate the relative strength of a score within an individual but, unlike normative measures, do not provide the absolute strength of a domain relative to a normative group. Examples of ipsative scores are the results of a forced choice measure.
Clinical Psychology
186. Beck's cognitive therapy includes all of the following characteristics except:
A it is referred to as "collaborative empiricism."
B it is structured and goal-oriented.
C relapse prevention is a focus throughout.
D it is based on an elaboration likelihood model.
ANSWER "D"
Beck's model includes all of the characteristics except an elaboration likelihood model, which is a cognitive model of attitude change that predicts that persuasion can occur in one of two ways--through a central and peripheral route (Petty, 1994). Beck's model is referred to as collaborative empiricism (answer A) because it is founded on a collaborative relationship between the therapist and client, and involves gathering evidence and testing hypotheses about the client's beliefs.
Ethics and Professional Practice
187. A managed health care organization decides that additional mental health services for one of your clients are not "medically necessary." You disagree with this conclusion, because you believe that the client may experience severe and long-term mental distress without at least two or three more counseling sessions. Based on current standards of practice as well as the implications of the court's ruling in Wickline v. California, your duty in this situation would be to:
A ensure that the client understands why you can no longer provide treatment and refer him to a place where he can obtain low-cost or free services.
B continue treating the client for a fee that is commensurate with his previous co-payment.
C appeal the decision of the managed health care organization.
D file a lawsuit against the managed health care organization.
ANSWER "C"
In the Wickline case, a patient sued a state-run managed care organization because it did not approve an eight-day post-surgical hospital stay and the patient later developed medical complications. The physician complied with the organization and released the patient without protest. The client lost the case against the organization, in part because the physician never appealed the organization's decision. The court held that a treating professional who "complies without protest with the limitations imposed by a third-party payor, when his [professional] judgment dictates otherwise, cannot avoid his ultimate responsibility for the patient's care." This case suggests that it is the health professional's obligation to protest in a situation such as the one described in the question, and a number of commentators suggest that advocacy for the patient's needs in such situations is an ethical obligation.
Developmental Psychology
188. According to Kohlberg, which of the following best describes moral reasoning and moral behavior:
A there is no relation between moral reasoning and moral action
B there is a positive relation between moral reasoning and moral action at all stages
C the strongest relation between moral reasoning and action is at the lower stages
D the strongest relation between moral reasoning and action is at the higher stages
ANSWER "D"
D. Kohlberg’s stage theory of moral development focuses on moral judgment and makes a distinction between moral judgment and moral action. Moral development is said to be enhanced by social experiences that produce cognitive conflict and provide the individual with an opportunity to take the perspective of others. Kohlberg proposes the higher the moral reasoning, the more likely moral action is and the greater consistency is between moral judgment and action because the higher stages increasingly employ more stable and general standards. (See: Kohlberg, L., & Candee, D. (1984). The relation of moral judgment to moral action. In W. Kurtines & J. L. Gewirtz (Eds.), Morality, moral behavior and moral development: Basic issues in theory and research, 52-73. New York: Wiley Interscience.) .
Ethics and Professional Practice
189. You have been seeing a female client in therapy for over a year and on your birthday she arrives with a beautifully gift-wrapped sweater and birthday card. You should
A explain that it is unethical and illegal to accept anything from a client.
B keep the card, but don't accept the sweater.
C explore the meaning of the gift.
D agree to keep the gift if the client will exchange it for a different color –you have already received quite a few green sweaters.
ANSWER "C"
None of the answers are quite right, but response C is the best choice. It suggests you first try to explore the clinical transference. However, once investigated, the resolution probably won't include a new sweater. Choice A is incorrect because while it is usually unethical to accept gifts, it is not illegal to do so. Choice B is a simple solution, but doesn't demonstrate any clinical exploration of the situation.
Neuropsychology
190. Acalculia, agraphia, difficulty distinguishing right from left, and finger agnosia are characteristic of
A Ganser's Syndrome
B Wernicke's Aphasia
C Gertsmann's Syndrome
D Kluver-Bucy Syndrome
ANSWER "C"
Gertsmann's syndrome is characterized by the symptoms listed in the question. It has been attributed to lesions of the parietal lobe, and it is sometimes observed in stroke victims.
Ethics and Professional Practice
191. You have been treating a 14-year-old female for the past year and during her final termination session she reports suicidal intent with a plan. You should:
A attempt to make a no-suicide contract
B immediately contact the parent(s)
C call the police
D not allow the client to leave your office until the risk has passed
ANSWER "B"
If a client poses an imminent danger to self, a breach of confidentiality, which could prevent the danger, is ethically and legally justified. This holds true for all clients, regardless of age. When the client is a minor, the most appropriate action would usually be to notify the client's parent(s). If they are not immediately available, you should then consider calling the police or PET. Attempting to make a no-suicide contract (A) would not be the most prudent option when there is suicidal intent with a plan, especially during a termination session. Not allowing the client to leave your office until the risk has passed (D) is not a realistic option.
Industrial/Organizational Psychology
192. Which of the following personality variables is significantly correlated with job performance across the widest variety of occupations?
A agreeableness
B emotional stability
C extraversion
D conscientiousness
ANSWER "D"
Factor analytic studies have identified five personality dimensions -- agreeableness, emotional stability, extraversion, openness, and conscientiousness -- that appear to underlie all other personality traits. The dimensions are commonly referred to in the literature "The Big Five." Of these, conscientiousness has the highest correlation with job performance and training success across a wide range of settings.
Neuropsychology
193. When an individual imagines movement or engaging in a motor activity, which part of the brain is most likely to be active?
A parietal lobe
B frontal lobe
C occipital lobe
D temporal lobe
ANSWER "A"
A. Studies on motor imagination indicate that the areas of the brain that are active during imagined motor activity are the same as when the activity is engaged in. Depending on the specific nature of the motor activity being imagined, differing areas are activated. The areas most likely to be active include the cerebellum, basal ganglia, the supplementary motor area and the parietal lobe, especially the somatosensory cortex.
School Psychology/Intelligence
194. Standardized ratings of adaptive behavior, such as the Adaptive Behavior Scale and the Vineland, are completed by caretakers, teachers, trained observers, etc., and measure:
A abilities and competencies.
B competencies, but not necessarily abilities.
C abilities, but not necessarily competencies.
D neither abilities nor competencies.
ANSWER "B"
The idea behind these rating instruments is that we are determining what the person actually does in an average, expectable environment. That is, what his or her competencies are: can he get dressed by himself, can she eat appropriately, does he engage in conversation when addressed, etc.?.
Clinical Psychology
195. In working with a triangulated family, a structural family therapist would
A work with the most differentiated family members in order to reduce their emotional reactivity.
B use a paradoxical intervention by taking steps to increase the family's initial level of triangulation.
C train the family in communication skills and techniques.
D upset the family's balance by blocking the family from its normal interactional pattern.
ANSWER "D"
Minuchin, the founder of structural family therapy, defines triangulation as a family boundary problem in which each parent demands that the child side with him or her against the other parent. According to Minuchin, the therapist's goal in this and other situations in which parents attempt to deflect conflict onto children is to restructure the parent-child and spousal subsystems so they function correctly. This involves introducing some stress into the family system to upset the family's balance and block the family from its normal way of interacting. For example, Minuchin describes strategies such as prohibiting children from talking about symptomatic behavior with their parents, rewarding parents for their capacity to nurture and support each other, and restructuring the parents in a coalition against their child.
Industrial/Organizational Psychology
196. The Strong Interest Inventory is most valid for predicting:
A Occupational success
B Occupational satisfaction
C Occupational temperament
D Occupational strength
ANSWER "B"
The Strong is most valid for predicting occupational choice and satisfaction. Like any other interest inventory, it is not valid for predicting occupational success.
Clinical Psychology
197. Which of the following best describes the underlying assumptions of the use of cognitive-behavioral techniques for pain management?
A Since pain, to a large extent, is due to tension, it is important to incorporate relaxation techniques into treatment.
B Chronic pain is connected to early childhood experiences, such as modeling of the sick role by parents.
C The environmental contingencies associated with the display of symptoms and the person's perception of control over symptoms must be attended to.
D Chronic pain has a variety of social, cultural, and economic causes.
ANSWER "C"
You probably could have answered this question just by using what you know about cognitive therapy and behavioral therapy -- you did not really need to know much about pain management. Behavioral treatments for managing pain tend to emphasize the importance of environmental contingencies (e.g., avoidance of work, sympathy) in maintaining pain, while cognitive techniques focus on the patient's beliefs -- particularly beliefs about control over his or her own life and therefore over his or her pain. Of course, this is consistent with the principles of cognitive-behavioral treatments in general.
Clinical Psychology
198. In terms of worldview, Sue and Sue (1990) propose that minority groups are increasingly more likely to exhibit an
A external locus of control and responsibility.
B internal locus of control and responsibility.
C external locus of control, internal locus of responsibility.
D internal locus of control, external locus of responsibility.
ANSWER "D"
Sue and Sue (Counseling the Culturally Different, New York, John Wiley, 1990) indicate that minority group members are becoming increasingly aware of their own ethnic identity and adopting a worldview that reflects an internal locus of control and an external locus of responsibility. An external locus of control and responsibility (answer A) is more characteristic of a "placater" who adopts a passive role. Internal locus of control and responsibility (answer B) is characteristic of the Westernized approach to counseling and white middle-class clients. An external locus of control and internal locus of responsibility (answer C) is more characteristic of "marginalized" individuals who have been oppressed by the dominant group.
Developmental Psychology
199. According to Piaget, which of the following underlies cognitive development?
A biological maturation
B biological maturation and environmental stimulation
C social interactions
D changes in the ability to process, store, and retrieve information
ANSWER "B"
Piaget proposed that cognitive development is the result of a combination of biological maturation and exposure to appropriate environmental stimuli. An implication of this assumption is that, even when a child is biologically ready for cognitive growth, it will not occur unless the child is also .
Developmental Psychology
200. Children generally begin to recognize racial differences based on physical traits between:
A 1 and 2 years
B 3 and 4 years
C 5 and 9 years
D 7 and 12 years
ANSWER "B"
Children's ethnic perspective-taking ability (EPTA) has been evaluated and described in several stages. In the first stage, which develops between 3 and 4 years, children first begin to describe ethnicity in terms of physical traits (e.g., skin color, clothes, physical features). In the next stage, which occurs between 5 and 9 years, they become able to accurately apply ethnic labels to themselves and others and they rely on other objective cues such as language and food preference, as well as physical appearance in ethnic labeling. In the next stage, from 7-12 years, children express a social perspective of ethnicity, including prejudice. And between 10 and 15 years, they begin to immerse themselves into their ethnic group (S.M. Quintana, V.C. Ybarra, P. Gonzalez-Doupe, & Y. DeBaessa, Cross-Cultural Evaluation of Ethnic Perspective-Taking Ability: An Exploratory Investigation With U.S. Latino and Guatemalan Ladino Children. Cultural Diversity and Ethnic Minority Psychology, 2000, 6 (4), 334-351).